0 of 258 questions completed
Questions:
You have already completed the assessment before. Hence you can not start it again.
Assessment is loading…
You must sign in or sign up to start the assessment.
You must first complete the following:
0 of 258 questions answered correctly
Your time:
Time has elapsed
You have reached 0 of 0 point(s), (0)
Earned Point(s): 0 of 0, (0)
0 Essay(s) Pending (Possible Point(s): 0)
A 48-year-old female with a history of mild congestive heart failure (CHF) treated with furosemide presents to the emergency room (ER) for evaluation of 24 hours of epigastric pain, nausea, and vomiting after eating a large meal in a restaurant. Previously, the patient had experienced intermittent right upper quadrant pain after eating. On examination, the patient has a temperature of 98.5°F and a pulse of 100. Her examination is remarkable for epigastric tenderness to palpation, normal bowel sounds, and no rebound tenderness or guarding. Laboratory studies are as follows:
Which of the following is the most likely diagnosis?
A 50-year-old female presents to your office for evaluation of solid food dysphagia without weight loss. Symptoms have been present for 6 months and are progressive. The patient has had two episodes of near impaction, but copious water ingestion and repeated swallows allowed the food bolus to pass. She has never had to present to the ER for disimpaction. She drinks five to six beers
per day, loves spicy foods, and smokes a pack of cigarettes daily with a total lifetime history of 30 pack-years. She has had intermittent heartburn symptoms for years and has not sought treatment. She takes hydrochlorothiazide for hypertension. Review of symptoms reveals chronic cough. Physical examination is unremarkable. Upper endoscopy reveals a distal esophageal stricture with inflammatory changes. Esophageal biopsies reveal benign mucosa with chronic inflammation. Gastric biopsies are unremarkable. Helicobacter pylori testing is negative. What is the most likely etiology of the patient’s stricture?
A 42-year-old man presents to your clinic with a 1-week history of pain and inflammation involving his right first metatarsophalangeal (MTP) joint. He
describes the pain as sudden in onset and worse at night. He denies experiencing any fever or traumatic injury to the joint and states that he has never had this type of pain before. He denies any chronic medical conditions, any prior surgery, and any current medication use. Besides an erythematous and exquisitely tender right first MTP joint, the remainder of his physical examination is unremarkable. Which of the following interventions is most appropriate at this time for your patient’s condition?
A 64-year-old male with a history of hypertension and tobacco abuse presents for follow-up after a routine physical during which he was found to have 4–5 red blood cells (RBCs) per high-power field (HPF) on a screening urinalysis. The urinalysis was negative for leukocytes, nitrites, epithelial cells, and ketones. The patient denies any complaints and the review of systems is essentially negative. In detecting microscopic hematuria, which of the following is true?
A 53-year-old Black male, with a history of hypertension, hepatitis C, and newly diagnosed nonsmall cell lung cancer, undergoes his first round of chemotherapy, which includes cisplatin. You are called to see this patient 5 days into his hospitalization for oliguria and laboratory abnormalities. Other than the chemotherapy, he is receiving lansoprazole, acetaminophen, and an infusion of D5— 0.9% normal saline at 50 mL/h. On examination, his BP is 98/60 and heart rate is irregular, between 40 and 50 bpm. His physical examination shows a middle-aged male in no acute distress. His cardiac examination is unremarkable, his lungs show bibasilar crackles, and the abdominal examination is positive for a palpable spleen tip without any hepatomegaly or abdominal tenderness. He has
trace bilateral ankle edema. His distal pulses are irregular. The neurologic examination was unremarkable. His laboratory (serum sample) results are as follows:
What is the most likely etiology of this patient’s acute renal failure?
A 72-year-old diabetic is transferred to your hospital for fever and altered mental status in the late summer. Symptoms started in this patient 1 week prior to admission. On physical examination, the patient was disoriented. There were no focal neurologic findings. There was a fine rash on the patient’s trunk. On oral examination, there were tongue fasciculations. A lumbar puncture was performed
which showed a glucose of 71 and a protein of 94; microscopy of the cerebrospinal fluid (CSF) revealed 9 RBC and 14 WBC (21 P, 68 L, 11 H). The
creatinine phosphokinase was 506. An electroencephalogram and MRI of the brain were normal. What further diagnostic test is the most appropriate?
A 45-year-old male with type II diabetes, hypertension, and hyperlipidemia presents to your clinic as a new patient. He has been out of his cholesterol medications and came to your office requesting a refill. The patient brought his most recent lipid profile (done after he was off his cholesterol medication for 3 months) which revealed:
Cholesterol (total): 242 mg/dL
HDL cholesterol: 38 mg/dL
Triglycerides (TGs): 660 mg/dL
LDL cholesterol = unable to calculate due
to high TGs
He also had recent liver function tests that were normal. Based on Adult Treatment Panel (ATP) III guidelines, which of the following medications should be the initial pharmacologic treatment for this patient?
A 29-year-old woman complains of fatigue and decreased exercise tolerance. She takes no medications and denies changes in the color of the stool. Physical examination is significant for pale skin and conjunctivae. Stool was negative for blood. Laboratory evaluation revealed Hgb of 7.8 g/dL, reticulocytopenia, microcytosis, and hypochromia. Which of the following is the most appropriate
next step in the management of the anemia in this woman?
A 23-year-old pregnant woman with type 1 diabetes was admitted to the Obstetrics service for DKA. The DKA was appropriately treated and has resolved. You were consulted for medical management of the diabetes, as her sugars have been labile throughout the hospital stay. Your history and review of records reveals that the patient has a long-standing history of non-compliance with diet and medication regimens. She currently uses any insulin she can get and does not eat regular meals. She has fluctuating blood sugars with episodes of hypoglycemia. You counsel the patient extensively, order nutrition and diabetic teaching consults, and discuss keeping home glucose logs. Assuming the patient will follow your advice, which regimen would you recommend to minimize fluctuating glucose readings?
A 45-year-old male has received intravenous contrast dye prior to CT scan of the abdomen. Twenty minutes later the patient reports severe pruritus. He denies respiratory distress, syncope, or palpitations. His blood pressure is 98/54, pulse is 90, and respiratory rate is 22. On physical examination, he has widespread
urticaria. His lungs are clear to auscultation. The next appropriate step would be which of the following?
A 19-year-old male who moved to your city 3 months ago comes to your office complaining of dry cough for the past 2–3 months. Along with the cough, he has had some shortness of breath with exertion. He denies fever, chills, nausea, vomiting, wheezing, and sneezing. The cough occurs mostly in the morning and improves as the day goes on. He denies similar complaints in the past and has no history of allergies. He says that his father had eczema and an allergy to eggs. What is the single best treatment for preventing symptoms in this patient?
A 25-year-old woman presents to your office complaining of cold hands. She describes them turning white as she reaches for orange juice in the frozen food section of the supermarket. It seems to be getting worse lately. She has no other symptoms but does note that she and her husband are contemplating pregnancy. Her examination today is unremarkable. In this patient, which of the following studies would be most likely to describe an increased risk of future systemic disease?
A 17-year-old male presents for evaluation of shortness of breath. He has episodes where he will audibly wheeze and have chest tightness. His symptoms
worsen if he tries to exercise, especially when it is cold. He has used an OTC inhaler with good relief of his symptoms, but he finds that his symptoms are
worsening. He now has episodes of wheezing on a daily basis and will have nighttime wheezing and coughing, on average, five or six times a month. You suspect a diagnosis of asthma. Which of the following would confirm your suspicion of the diagnosis of asthma?
A 56-year-old Black male construction worker comes for evaluation of a worsening, nonproductive cough that he first noticed 2 months before. During the last week the cough has worsened and has become productive of yellow, blood tinged sputum. He reports his appetite is poor, and he has lost approximately 15 lbs over the past 2 months. You take a social history and find out he has smoked two packs of cigarettes a day since he was 16 years old. He states that he drinks approximately 10 beers per week. You perform a physical examination. He appears chronically ill; however, his vital signs are normal. The head and neck examination is within normal limits. There are decreased breath sounds in the left upper chest. Breath sounds are distant in the other lung fields. The diaphragms are low. There is no palpable hepatosplenomegaly. You order a posterior-to-anterior (PA) and lateral CXR. The chest radiogram shows opacity of the left upper lobe. There are no pleural effusions. The cardiac silhouette is not enlarged. The mediastinum does not appear enlarged. The patient has the follow-up test that you recommend. It shows a 5-cm mass compressing the left upper lobe bronchus with consolidation of the left upper lobe. Two 1 cm peribronchial lymph nodes near the left main stem bronchus and several 1.5–2.0 cm mediastinal lymph
nodes are seen. The hilar nodes do not appear enlarged. There are no enlarged lymph nodes visualized in the right chest. There are no lesions seen in the right lung. There are emphysematous changes involving both lungs. A biopsy of the lung mass shows a small cell carcinoma. What should be done next?
A 48-year-old woman with metastatic breast cancer presents to the Emergency Center complaining of a 4-day history of nausea, anorexia, and generalized weakness. Her husband reports that she has been more somnolent, sleeping 12–14 hours per day, and at times she seems confused. CT scan of the brain reveals no abnormalities. Initial laboratory evaluation reveals a normal CBC, but her BUN is elevated at 32 mg/dL with a slight elevation of serum creatinine above her baseline. Her serum calcium is elevated at 15 mg/dL. What is the most likely cause of her hypercalcemia?
A 32-year-old male is seen in the ER with a nondisplaced fracture of the ulna after a fall. Incidentally, the intern noticed that the patient is jaundiced and has a palpable spleen. He orders a CBC which shows a Hgb of 10.2 g/dL. The patient reveals that he has chronic anemia and intermittently has been prescribed iron.
On further questioning, he says that he had a cholecystectomy at age 23 and that he has family members with similar symptoms. The intern reviews the peripheral smear and finds spherocytes. What would be the best way to confirm this man’s diagnosis?
A 48-year-old female with a history of mild congestive heart failure (CHF) treated with furosemide presents to the emergency room (ER) for evaluation of 24 hours of epigastric pain, nausea, and vomiting after eating a large meal in a restaurant. Previously, the patient had experienced intermittent right upper quadrant pain after eating. On examination, the patient has a temperature of 98.5°F and a pulse of 100. Her examination is remarkable for epigastric tenderness to palpation, normal bowel sounds, and no rebound tenderness or guarding. Laboratory studies are as follows:
What would the most appropriate next test to order be?
A 50-year-old female presents to your office for evaluation of solid food dysphagia without weight loss. Symptoms have been present for 6 months and are progressive. The patient has had two episodes of near impaction, but copious water ingestion and repeated swallows allowed the food bolus to pass. She has never had to present to the ER for disimpaction. She drinks five to six beers
per day, loves spicy foods, and smokes a pack of cigarettes daily with a total lifetime history of 30 pack-years. She has had intermittent heartburn symptoms for years and has not sought treatment. She takes hydrochlorothiazide for hypertension. Review of symptoms reveals chronic cough. Physical examination is unremarkable. Upper endoscopy reveals a distal esophageal stricture with inflammatory changes. Esophageal biopsies reveal benign mucosa with chronic inflammation. Gastric biopsies are unremarkable. Helicobacter pylori testing is negative. What is the next best step in therapy for this patient?
A 42-year-old man presents to your clinic with a 1-week history of pain and inflammation involving his right first metatarsophalangeal (MTP) joint. He
describes the pain as sudden in onset and worse at night. He denies experiencing any fever or traumatic injury to the joint and states that he has never had this type of pain before. He denies any chronic medical conditions, any prior surgery, and any current medication use. Besides an erythematous and exquisitely tender right first MTP joint, the remainder of his physical examination is unremarkable. What is a potential long-term complication of this patient’s condition?
A 52-year-old man presents to the ED with a complaint of rectal bleeding and hematuria. He has a medical history significant for atrial fibrillation diagnosed 10 years ago and states that he takes metoprolol as well as warfarin for this condition. Upon examination, you find that his blood pressure is 122/78, his pulse is 84, his respiratory rate is 18, and his O₂ saturation is 98% on room air. He has an irregularly irregular heart rhythm, gingival bleeding, and some bruises on his extremities. He has a positive fecal occult blood test, and laboratory studies return showing an international normalized ratio (INR) of 16.5. You order that the patient’s warfarin be held. Which of the following is the most appropriate additional intervention at this time?
A 53-year-old Black male, with a history of hypertension, hepatitis C, and newly diagnosed nonsmall cell lung cancer, undergoes his first round of chemotherapy, which includes cisplatin. You are called to see this patient 5 days into his hospitalization for oliguria and laboratory abnormalities. Other than the chemotherapy, he is receiving lansoprazole, acetaminophen, and an infusion of D5— 0.9% normal saline at 50 mL/h. On examination, his BP is 98/60 and heart rate is irregular, between 40 and 50 bpm. His physical examination shows a middle-aged male in no acute distress. His cardiac examination is unremarkable, his lungs show bibasilar crackles, and the abdominal examination is positive for a palpable spleen tip without any hepatomegaly or abdominal tenderness. He has
trace bilateral ankle edema. His distal pulses are irregular. The neurologic examination was unremarkable. His laboratory (serum sample) results are as follows:
What would be the most likely finding on this patient’s ECG?
A 24-year-old male presents with sore throat, subjective fever, abdominal pain, and bad breath. He says that a neighbor’s child is currently being treated for strep throat. On examination, his temperature is 101.1°F and his other vital signs are normal. He appears well. His throat is erythematous and his tonsils are enlarged, but there are no pharyngeal or tonsillar exudates. He has no cervical adenopathy.
He has an occasional cough but his lungs are clear. His abdominal examination is normal. The presence of which of the following findings is a clinical predictor for the diagnosis of streptococcal pharyngitis?
A 48-year-old woman complaining of dysuria is diagnosed with a UTI by urinalysis. Urine culture and sensitivities reveal that the causative organism
belongs to the genus Klebsiella and is resistant to multiple antibiotics. Based upon the results available, you decide to begin therapy with gentamicin. Before doing so, you explain to the patient that antibiotics such as gentamicin are often associated with which of the following?
A 23-year-old woman presents to your acute care clinic with a complaint of fever, sore throat, and malaise of sudden onset. Her prior medical history is significant for schizophrenia. Her vitals signs are: BP 116/80, HR 112, RR 26, Temp 100.6 degrees Fahrenheit. On physical examination, her oral cavity features painful aphthous ulcers as well as swollen gums. Initial laboratory testing includes a CBC
which returns with the following results:
• Leukocyte count 800/mm³
• Hgb 12.1 g/dL
• HCT 37.0%
• Platelet count 212 × 10⁹/L
• Differential:
• Neutrophils, segmented 52%
• Neutrophils, bands 3%
• Lymphocytes 35%
• Monocytes 7%
• Eosinophils 3%
• Basophils 0%
Use of which of the following must be considered when formulating a differential diagnosis to explain this patient’s symptoms?
A 54-year-old man presents with a 3-cm right thyroid nodule that was found incidentally by the patient while shaving. He denies any pain or discomfort. He denies any history of thyroid disease, any family history of thyroid disease, and any history of head/neck irradiation. He notes a 10-lb weight loss over the past
6 months. His examination is only remarkable for the firm right thyroid nodule. The remainder of the thyroid is not palpable. There is no adenopathy. Heart rate is 90/minute and regular. The skin is warm and moist, and a fine tremor is present when he holds his hands out. TSH level is <.02 μU/mL. Which of the following is the most appropriate next step?
A 45-year-old male has received intravenous contrast dye prior to CT scan of the abdomen. Twenty minutes later the patient reports severe pruritus. He denies respiratory distress, syncope, or palpitations. His blood pressure is 98/54, pulse is 90, and respiratory rate is 22. On physical examination, he has widespread
urticaria. His lungs are clear to auscultation. This intervention works well in cases such as this due to activity directed toward which of these receptors?
A 19-year-old male who moved to your city 3 months ago comes to your office complaining of dry cough for the past 2–3 months. Along with the cough, he has had some shortness of breath with exertion. He denies fever, chills, nausea, vomiting, wheezing, and sneezing. The cough occurs mostly in the morning and improves as the day goes on. He denies similar complaints in the past and has no history of allergies. He says that his father had eczema and an allergy to eggs. Which of the following properties of albuterol makes it a more appropriate choice than epinephrine for relief of acute episodes of bronchoconstriction?
A 25-year-old woman presents to your office complaining of cold hands. She describes them turning white as she reaches for orange juice in the frozen food section of the supermarket. It seems to be getting worse lately. She has no other symptoms but does note that she and her husband are contemplating pregnancy. Her examination today is unremarkable. Which of the following antibodies can cross the placenta and cause the syndrome of neonatal lupus?
A 17-year-old male presents for evaluation of shortness of breath. He has episodes where he will audibly wheeze and have chest tightness. His symptoms
worsen if he tries to exercise, especially when it is cold. He has used an OTC inhaler with good relief of his symptoms, but he finds that his symptoms are
worsening. He now has episodes of wheezing on a daily basis and will have nighttime wheezing and coughing, on average, five or six times a month. You suspect a diagnosis of asthma. Your diagnostic workup confirms the diagnosis of asthma. What clinical classification of asthma does this patient have?
A 55-year-old woman presents to your office with painful hands, causing difficulty opening jars and turning the key in the ignition of her car. She is fatigued and she notices joint stiffness, but limbers up by lunch. She has trouble getting her rings off because of enlarging knuckles. About a year ago, she tried some OTC ibuprofen, which seemed to help, but led to the development of a bleeding ulcer
severe enough to require transfusion and ICU care. Otherwise, her health is good, and her review of systems is negative. Your physical examination reveals tenderness and swelling at the index proximal interphalangeal and metacarpophalangeal joints bilaterally. There are small effusions on both knees. She has tenderness to lateral compression of the forefoot area bilaterally. The following data are obtained: normal CBC; normal basic metabolic panel; ESR 40 mm/h; ALT 90 U/L; AST 110 U/L; alkaline phosphatase 70 U/L; bilirubin 0.2 mg/dL; uric acid 5.1 mg/dL; urinalysis is normal. ACE level is normal. Rheumatoid factor is 60 and ANA is positive 1:40 speckled pattern. The next most important test would be which of the following?
A 64-year-old man with a long history of smoking but no significant past medical history presents to his physician’s office complaining of a 1–2 week history of worsening dyspnea, facial swelling, and discoloration. He has also had swelling and discomfort of his right arm. Physical examination reveals edema with plethora of the face and right arm, with prominent collateral veins over the chest wall. What test is most likely to establish the cause of the patient’s symptoms?
A 74-year-old female with a history of hypertension and hypothyroidism is admitted with easy bruising, guaiac positive stools, and anemia (Hgb 8.1 g/dL). Screening coagulation tests reveal a prolonged activated partial thromboplastin time (aPTT) with a normal prothrombin time (PT) and platelet count. What is the
next step in the diagnosis of this woman’s problem?
A 48-year-old female with a history of mild congestive heart failure (CHF) treated with furosemide presents to the emergency room (ER) for evaluation of 24 hours of epigastric pain, nausea, and vomiting after eating a large meal in a restaurant. Previously, the patient had experienced intermittent right upper quadrant pain after eating. On examination, the patient has a temperature of 98.5°F and a pulse of 100. Her examination is remarkable for epigastric tenderness to palpation, normal bowel sounds, and no rebound tenderness or guarding. Laboratory studies are as follows:
The patient is made NPO (nothing by mouth) and vigorously hydrated. After 3 days, the amylase and lipase normalize, but the bilirubin rises to 4.2 mg/dL. An endoscopic retrograde cholangiopancreatography (ERCP) is performed, and the following cholangiogram is obtained (see Figure below). What is the best treatment option at this time?
A 50-year-old female presents to your office for evaluation of solid food dysphagia without weight loss. Symptoms have been present for 6 months and are progressive. The patient has had two episodes of near impaction, but copious water ingestion and repeated swallows allowed the food bolus to pass. She has never had to present to the ER for disimpaction. She drinks five to six beers
per day, loves spicy foods, and smokes a pack of cigarettes daily with a total lifetime history of 30 pack-years. She has had intermittent heartburn symptoms for years and has not sought treatment. She takes hydrochlorothiazide for hypertension. Review of symptoms reveals chronic cough. Physical examination is unremarkable. Upper endoscopy reveals a distal esophageal stricture with inflammatory changes. Esophageal biopsies reveal benign mucosa with chronic inflammation. Gastric biopsies are unremarkable. Helicobacter pylori testing is negative. The patient is at increased risk for which of the following illnesses?
A 42-year-old man presents to your clinic with a 1-week history of pain and inflammation involving his right first metatarsophalangeal (MTP) joint. He
describes the pain as sudden in onset and worse at night. He denies experiencing any fever or traumatic injury to the joint and states that he has never had this type of pain before. He denies any chronic medical conditions, any prior surgery, and any current medication use. Besides an erythematous and exquisitely tender right first MTP joint, the remainder of his physical examination is unremarkable. After 1 week of treatment, your patient states that his pain and inflammation have resolved. You measure a serum urate level and find it elevated. Urinary urate excretion is high. Which of the following interventions is now most appropriate?
A 52-year-old man presents to the ED with a complaint of rectal bleeding and hematuria. He has a medical history significant for atrial fibrillation diagnosed 10 years ago and states that he takes metoprolol as well as warfarin for this condition. Upon examination, you find that his blood pressure is 122/78, his pulse is 84, his respiratory rate is 18, and his O₂ saturation is 98% on room air. He has an irregularly irregular heart rhythm, gingival bleeding, and some bruises on his extremities. He has a positive fecal occult blood test, and laboratory studies return showing an international normalized ratio (INR) of 16.5. Reviewing the patient’s medication list, you note that he mentions a “medication for depression” but does not recall the name. Which of the following is most concerning given the patient’s current condition?
A 53-year-old Black male, with a history of hypertension, hepatitis C, and newly diagnosed nonsmall cell lung cancer, undergoes his first round of chemotherapy, which includes cisplatin. You are called to see this patient 5 days into his hospitalization for oliguria and laboratory abnormalities. Other than the chemotherapy, he is receiving lansoprazole, acetaminophen, and an infusion of D5— 0.9% normal saline at 50 mL/h. On examination, his BP is 98/60 and heart rate is irregular, between 40 and 50 bpm. His physical examination shows a middle-aged male in no acute distress. His cardiac examination is unremarkable, his lungs show bibasilar crackles, and the abdominal examination is positive for a palpable spleen tip without any hepatomegaly or abdominal tenderness. He has
trace bilateral ankle edema. His distal pulses are irregular. The neurologic examination was unremarkable. His laboratory (serum sample) results are as follows:
Which of the following would be a part of the IMMEDIATE treatment strategy in this patient?
A 24-year-old male presents with sore throat, subjective fever, abdominal pain, and bad breath. He says that a neighbor’s child is currently being treated for strep throat. On examination, his temperature is 101.1°F and his other vital signs are normal. He appears well. His throat is erythematous and his tonsils are enlarged, but there are no pharyngeal or tonsillar exudates. He has no cervical adenopathy.
He has an occasional cough but his lungs are clear. His abdominal examination is normal. Which of the following is the recommended first-line agent for the treatment of group A streptococcal pharyngitis?
A 48-year-old woman complaining of dysuria is diagnosed with a UTI by urinalysis. Urine culture and sensitivities reveal that the causative organism
belongs to the genus Klebsiella and is resistant to multiple antibiotics. Based upon the results available, you decide to begin therapy with gentamicin. Which of the following irreversible complications is also associated with gentamicin use?
A 23-year-old woman presents to your acute care clinic with a complaint of fever, sore throat, and malaise of sudden onset. Her prior medical history is significant for schizophrenia. Her vitals signs are: BP 116/80, HR 112, RR 26, Temp 100.6 degrees Fahrenheit. On physical examination, her oral cavity features painful aphthous ulcers as well as swollen gums. Initial laboratory testing includes a CBC
which returns with the following results:
• Leukocyte count 800/mm³
• Hgb 12.1 g/dL
• HCT 37.0%
• Platelet count 212 × 10⁹/L
• Differential:
• Neutrophils, segmented 52%
• Neutrophils, bands 3%
• Lymphocytes 35%
• Monocytes 7%
• Eosinophils 3%
• Basophils 0%
Which of the following best describes the expected course of the patient’s condition?
A 19-year-old woman who is 2 months post-partum complains of palpitations, heat intolerance, tremulousness, weight loss, and fatigue. Her thyroid is prominent and firm but non-tender. Serum TSH level was undetectable. A
nuclear medicine radioactive iodine uptake is performed and shows no uptake of iodine in the neck. Which of the following is the most appropriate next step?
A 20-year-old male has had a recent wide local excision of a 1.5 mm melanoma from the right ankle. There is no evidence of metastatic disease. The most important prognostic factor for this patient is which of the following?
A 63-year-old male presents to your office with palpitations for the past 3 weeks. He has had no chest pains or dyspnea. He has no significant medical history and takes no medications. He does not smoke cigarettes and a recent lipid panel was normal. On examination, he is in no apparent distress. His pulse is 115 bpm and irregular. His BP is 125/77. His lungs are clear and his cardiac examination reveals an irregularly irregular rhythm with no murmurs, rubs, or gallops. Which of the following is most likely to be found on an ECG?
A 55-year-old woman presents to your office with painful hands, causing difficulty opening jars and turning the key in the ignition of her car. She is fatigued and she notices joint stiffness, but limbers up by lunch. She has trouble getting her rings off because of enlarging knuckles. About a year ago, she tried some OTC ibuprofen, which seemed to help, but led to the development of a bleeding ulcer
severe enough to require transfusion and ICU care. Otherwise, her health is good, and her review of systems is negative. Your physical examination reveals tenderness and swelling at the index proximal interphalangeal and metacarpophalangeal joints bilaterally. There are small effusions on both knees. She has tenderness to lateral compression of the forefoot area bilaterally. Which of the following tests is most likely to result in a diagnosis?
A 17-year-old male presents for evaluation of shortness of breath. He has episodes where he will audibly wheeze and have chest tightness. His symptoms
worsen if he tries to exercise, especially when it is cold. He has used an OTC inhaler with good relief of his symptoms, but he finds that his symptoms are
worsening. He now has episodes of wheezing on a daily basis and will have nighttime wheezing and coughing, on average, five or six times a month. You suspect a diagnosis of asthma. Which of the following is the most appropriate
pharmacologic regimen for this patient?
A 55-year-old woman presents to your office with painful hands, causing difficulty opening jars and turning the key in the ignition of her car. She is fatigued and she notices joint stiffness, but limbers up by lunch. She has trouble getting her rings off because of enlarging knuckles. About a year ago, she tried some OTC ibuprofen, which seemed to help, but led to the development of a bleeding ulcer
severe enough to require transfusion and ICU care. Otherwise, her health is good, and her review of systems is negative. Your physical examination reveals tenderness and swelling at the index proximal interphalangeal and metacarpophalangeal joints bilaterally. There are small effusions on both knees. She has tenderness to lateral compression of the forefoot area bilaterally. The test ordered above is negative and an anticyclic citrullinated peptide (anti-CCP) antibody is strongly positive (600). Which of the following is the probable source of her symptoms?
A 64-year-old man with a long history of smoking but no significant past medical history presents to his physician’s office complaining of a 1–2 week history of worsening dyspnea, facial swelling, and discoloration. He has also had swelling and discomfort of his right arm. Physical examination reveals edema with plethora of the face and right arm, with prominent collateral veins over the chest wall. Following the establishment of a diagnosis, what is the most appropriate initial therapy?
A 23-year-old African-American presents with acute-onset pain in the abdomen, back, and legs. On physical examination, his pulse is 115 bpm, respiratory rate is 20, blood pressure is 100/70 mmHg, and temperature is 101°F. There is scleral icterus, a systolic ejection murmur at the right upper sternal border, bilateral rhonchi, a right upper quadrant abdominal scar from a cholecystectomy, and a diffusely tender abdomen without rebound. A neurologic examination is normal. A peripheral blood smear is shown in Figure.
Among the initial orders for this patient should be which of the following?
Your patient who was recently prescribed an antibiotic returns to your clinic for a
follow-up visit. Although she was feeling better, the instructions on the bottle were to take the medication for total of 10 days. She wants to know if she still has to take the medication three times daily as she has improved. You explain to her that the dosing regimen is based on the biological half-life of a drug, which is generally related to which of the following?
A 65-year-old man presents to your office for evaluation of abdominal pain. The patient states that he has epigastric pain that radiates to his back. The pain is worse with eating and improves with fasting. The pain has been present for 6 months and is gradually worsening. The patient has lost 15 lbs but feels his oral
intake has been adequate. He complains of greasy stools and frequent thirst and urination. Examination reveals a thin male with temporal wasting and moderate abdominal pain with palpation. The patient consumes approximately 10–15 beers per day and smokes a pack of cigarettes per day for the past 20 years. What would be the best initial test to do in this patient?
Which of the following produces the greatest increase in bone mineral density (BMD) in patients with osteoporosis?
A 72-year-old African American male presents for a routine health examination. He states that he would like to have a “screening for cancer.” In the United States, based on his sex, race, and age, what is the most likely malignancy for this patient?
A 53-year-old White female, with a history of systemic lupus erythematosus (SLE), hypertension, and peripheral vascular disease, is admitted to the hospital for chest pain and dyspnea. Her cardiac enzymes were positive for acute MI. She subsequently undergoes a cardiac catheterization and stenting of the right coronary artery. Her postcardiac catheterization course is unremarkable, and she is discharged home 3 days later with adequate blood pressure control. Five days later, she is brought to the ER by her husband for abdominal pain and nausea. Her medications consist of aspirin, metoprolol, and prednisone. On physical examination, her blood pressure is 190/95 and her heart rate is 85 bpm. In general, she appears nauseated but is in no acute distress. Her cardiac examination reveals a regular rate and rhythm without murmur or rub. Her lung fields are clear bilaterally. The abdominal examination is positive for diffuse discomfort, without guarding or rebound, and normoactive bowel sounds; her stool is positive for occult blood. Her lower extremities have trace edema bilaterally with 2+ distal pulses; moreover, she has a reddish-blue discoloration on both her lower extremities. You retrieve her records from prior hospitalization. The
patient’s laboratory results are as follows:
What is the most likely cause of this patient’s acute renal failure?
A 22-year-old male presents to an acute care clinic in order to have two genital lesions evaluated. He first noticed the lesions about 2 weeks ago, but delayed
seeking medical care because he believed they were harmless due to the lack of any discomfort. He states that he does engage in unprotected sexual intercourse, with the most recent time being 1 month ago. On examination, the glans penis features two distinct nontender papules with elevated edges surrounding ulcerated craters. They each measure 1 cm in diameter. There is also nontender bilateral inguinal lymphadenopathy. Which of the following is true about this patient’s condition?
A 48-year-old woman complaining of dysuria is diagnosed with a UTI by urinalysis. Urine culture and sensitivities reveal that the causative organism
belongs to the genus Klebsiella and is resistant to multiple antibiotics. Based upon the results available, you decide to begin therapy with gentamicin. Which of the following would lead to the classification of this patient’s infection as
“complicated?”
A 62-year-old female with a history of a recent pulmonary embolus presents to your office for follow-up on anticoagulation treatment. She takes warfarin on a daily basis. She reports that for the last week she has noticed mild rectal
bleeding and multiple bruises over the extremities with minimal trauma. She is comfortable appearing with normal vital signs and is not orthostatic. You ordered a stat CBC and PT/INR which revealed a mildly decreased Hgb at 11 g/dL and an elevated INR of 7. Which of the following would be the most appropriate intervention?
A 60-year-old man with a history of severe chronic obstructive pulmonary disease
(COPD), which is steroid dependent, is admitted to the ICU with pulmonary infiltrates and a sepsis syndrome. His hospital course is complicated by acute renal insufficiency and respiratory failure. Therapy includes glucocorticoids
and dopamine. He has no history of thyroid disease. Several weeks into his hospital course, the following laboratory studies are performed:
Based on these laboratory studies, which of the following is the most appropriate next step?
A 20-year-old male has had a recent wide local excision of a 1.5 mm melanoma from the right ankle. There is no evidence of metastatic disease. Which of the following interventions is most appropriate in addition to wide local excision of the patient’s melanoma?
A 63-year-old male presents to your office with palpitations for the past 3 weeks. He has had no chest pains or dyspnea. He has no significant medical history and takes no medications. He does not smoke cigarettes and a recent lipid panel was normal. On examination, he is in no apparent distress. His pulse is 115 bpm and irregular. His BP is 125/77. His lungs are clear and his cardiac examination reveals an irregularly irregular rhythm with no murmurs, rubs, or gallops. An abnormal result of which of the following laboratory tests would be most likely to explain the cause of this condition?
An 18-year-old female presents for follow-up of a rash. She states that she has been using OTC antifungal preparations without success. She used OTC cortisone cream with mild improvement, but the lesions never disappeared. Her condition also recurred with full severity once the cortisone cream was discontinued. Her family history reveals a first degree relative with similar rash. The rash is over the elbows, trunk, and scalp (see Figure).
Which other physical finding would be consistent with your presumptive diagnosis?
A 74-year-old male with a history of hypertension, CAD, and a 50 pack-year history of smoking presents with complaints of pain and cramping sensation of the thigh and buttock areas for the past 2 months. On detailed history, patient reports that the pain is usually during ambulation and relieves with sitting down. The pain does not change with respect to sitting or supine position. He denies any recent trauma, weakness of the legs, or paresthesias. He takes his prescription medications regularly and denies using alcohol, drugs, or any herbs/supplements. Which of the following should be performed as an initial test to help confirm
your clinical impression?
A 50-year-old man presents to your office with fatigue and weakness. He first noticed it a few weeks ago while trying to hang pictures with his wife. His legs have begun to ache as he walks up stairs. He has lost about 20 lbs in the last 3 months. Most recently, he has found that he is more constipated and has trouble rising from the commode. Your physical examination reveals modest proximal weakness, no articular swelling, rash, or any other pertinent findings. Blood work from a recent insurance examination revealed:
Sodium 142 meq/L; potassium 3.8 meq/L; chloride 107 meq/L; bicarbonate 29 meq/L; BUN 30 mg/dL; Cr 1.6 mg/dL; WBC 6.8; Hgb 13.6 g/dL; HCT 40%; MCV 88.0 μm3; platelets 240,000/mm3; AST 200 U/L; ALT 250 U/L; alkaline phosphatase 70 U/L; bilirubin 0.3 mg/dL; ESR 40 mm/h. Along with a creatine phosphokinase (CPK), which of the following tests should be ordered first?
A 25-year-old woman with acute myelocytic leukemia is undergoing induction chemotherapy and presents to the Emergency Center complaining of a 1-day history of fever to 102º F with no other symptoms. Other than having a fever, her vital signs and physical examination are normal. Laboratory evaluation reveals pancytopenia, with a WBC count of 0.3 k/μL, hemoglobin concentration of 9.2 mg/dL, and platelet count of 23,000/μL. What is the most appropriate initial management?
A 23-year-old African-American presents with acute-onset pain in the abdomen, back, and legs. On physical examination, his pulse is 115 bpm, respiratory rate is 20, blood pressure is 100/70 mmHg, and temperature is 101°F. There is scleral icterus, a systolic ejection murmur at the right upper sternal border, bilateral rhonchi, a right upper quadrant abdominal scar from a cholecystectomy, and a diffusely tender abdomen without rebound. A neurologic examination is normal. A peripheral blood smear is shown in Figure.
A CBC with differential and platelets later shows the Hgb to be 6.4, white blood count is 2100 with 85% polymorphonuclear forms, platelet count is 100, and the reticulocyte count is 0.5. Which of the following would be an appropriate next test?
A 45-year-old male comes to your office for his first annual checkup in the last 10 years. On first impression, he appears overweight but is otherwise healthy and has no specific complaints. He has a brother with diabetes and a sister with high blood pressure. Both of his parents are deceased and his father died of a stroke at age 73. He is a long-standing heavy smoker and only drinks alcohol on special
occasions. On physical examination, his blood pressure is 166/90 in the left arm and 164/88 in the right arm. The rest of the examination is unremarkable. He is concerned about his health and does not want to end up on medication, like his siblings. Regarding your initial recommendations, which of the following would be most appropriate?
A 35-year-old woman with a history of major depressive disorder is brought into the ED by her boyfriend. He believes she may have overdosed on pain medication in an effort to hurt herself. He gives you three medication bottles which he discovered empty and states that they were nearly full before leaving for work earlier that morning. However, the timing of the ingestion is unclear. All three of the bottles apparently held acetaminophen-containing medications. Examination of the patient reveals a tired-appearing woman complaining of nausea and right upper quadrant abdominal pain. Depletion of which of the following is primarily responsible for the hepatotoxicity being experienced by the patient?
A 65-year-old man presents to your office for evaluation of abdominal pain. The patient states that he has epigastric pain that radiates to his back. The pain is worse with eating and improves with fasting. The pain has been present for 6 months and is gradually worsening. The patient has lost 15 lbs but feels his oral
intake has been adequate. He complains of greasy stools and frequent thirst and urination. Examination reveals a thin male with temporal wasting and moderate abdominal pain with palpation. The patient consumes approximately 10–15 beers per day and smokes a pack of cigarettes per day for the past 20 years. On further questioning, the patient reports that he recently had a motor vehicle accident at
night because he felt he could not see clearly. The most likely cause of this symptom is which of the following?
A 23-year-old female graduate student with acne and asthma presents to you with a chief complaint of headaches. She has noted a gradual increase in the intensity and frequency of the headaches to the point where they are interfering with her daily activities and studies. Your examination shows an obese young lady with papilledema. The remainder of your physical examination is normal. Which of the following is the most appropriate management at this time?
A 72-year-old man comes to your clinic for the first time, accompanied by his wife. His wife states that she is concerned because he has been growing increasingly forgetful over the past year. Within the past month, he has forgotten to turn off the stove and has got lost while walking to the post office one block away from their home. His past medical history is significant for well-controlled diabetes and
chronic lower back pain. He has no history of falls or traumatic injury to the head. Examination of the patient is significant for a score of 18 on a Mini-Mental Status Examination (MMSE). During the administration of the MMSE, the patient blurts out that his wife brought him to the doctor because she is having an extramarital relationship. Which of the following accurately describes this patient’s condition?
A 53-year-old White female, with a history of systemic lupus erythematosus (SLE), hypertension, and peripheral vascular disease, is admitted to the hospital for chest pain and dyspnea. Her cardiac enzymes were positive for acute MI. She subsequently undergoes a cardiac catheterization and stenting of the right coronary artery. Her postcardiac catheterization course is unremarkable, and she is discharged home 3 days later with adequate blood pressure control. Five days later, she is brought to the ER by her husband for abdominal pain and nausea. Her medications consist of aspirin, metoprolol, and prednisone. On physical examination, her blood pressure is 190/95 and her heart rate is 85 bpm. In general, she appears nauseated but is in no acute distress. Her cardiac examination reveals a regular rate and rhythm without murmur or rub. Her lung fields are clear bilaterally. The abdominal examination is positive for diffuse discomfort, without guarding or rebound, and normoactive bowel sounds; her stool is positive for occult blood. Her lower extremities have trace edema bilaterally with 2+ distal pulses; moreover, she has a reddish-blue discoloration on both her lower extremities. You retrieve her records from prior hospitalization. The
patient’s laboratory results are as follows:
Which of the following tests is helpful in distinguishing volume depletion as a possible cause of acute renal failure?
A 22-year-old male presents to an acute care clinic in order to have two genital lesions evaluated. He first noticed the lesions about 2 weeks ago, but delayed
seeking medical care because he believed they were harmless due to the lack of any discomfort. He states that he does engage in unprotected sexual intercourse, with the most recent time being 1 month ago. On examination, the glans penis features two distinct nontender papules with elevated edges surrounding ulcerated craters. They each measure 1 cm in diameter. There is also nontender bilateral inguinal lymphadenopathy. The drug of choice for treating this patient works by which of the following mechanisms?
A 53-year-old fisherman develops pain and swelling of the right hand 8 hours after suffering a fish hook injury to the finger. On physical examination, the patient’s temperature is 102.8°F and the patient appears septic. The patient’s hand and a Gram stain of material aspirated from a bulla are shown in Figures (a) and (b).
(a)
(b)
What is the most likely etiology of this bacteremia?
A 42 year old male admitted for pulmonary embolus was placed on heparin, dosed by a weight based protocol. However, later in the day, you receive a call from the floor nurse stating that the patient had spontaneous epistaxis and a very high aPTT. Use of which of the following would be best at this time?
A 28-year-old woman presents for evaluation of primary infertility. She has had fewer than four periods per year since menarche at age 14, facial hirsutism, acne, and weight gain. On examination, she has a BP 150/100. Her body mass index (BMI) is 40. Acanthosis nigricans is noted along the posterior surface of her neck. Which of the following laboratory studies is most likely to be abnormal in this patient?
A 20-year-old male has had a recent wide local excision of a 1.5 mm melanoma from the right ankle. There is no evidence of metastatic disease.One year after his initial diagnosis and treatment, the patient develops a palpable right inguinal lymph node. Inguinal lymph node dissection reveals one node positive for metastatic melanoma; the remaining nodes are negative. A complete restaging workup shows no evidence of any additional metastatic disease. What is the correct stage for this patient?
A 63-year-old male presents to your office with palpitations for the past 3 weeks. He has had no chest pains or dyspnea. He has no significant medical history and takes no medications. He does not smoke cigarettes and a recent lipid panel was normal. On examination, he is in no apparent distress. His pulse is 115 bpm and irregular. His BP is 125/77. His lungs are clear and his cardiac examination reveals an irregularly irregular rhythm with no murmurs, rubs, or gallops. Which of the following studies would be most appropriate to order at this time?
An 18-year-old female presents for follow-up of a rash. She states that she has been using OTC antifungal preparations without success. She used OTC cortisone cream with mild improvement, but the lesions never disappeared. Her condition also recurred with full severity once the cortisone cream was discontinued. Her family history reveals a first degree relative with similar rash. The rash is over the elbows, trunk, and scalp (see Figure).
Which of the following tests is most likely to confirm your clinical suspicion?
A 74-year-old male with a history of hypertension, CAD, and a 50 pack-year history of smoking presents with complaints of pain and cramping sensation of the thigh and buttock areas for the past 2 months. On detailed history, patient reports that the pain is usually during ambulation and relieves with sitting down. The pain does not change with respect to sitting or supine position. He denies any recent trauma, weakness of the legs, or paresthesias. He takes his prescription medications regularly and denies using alcohol, drugs, or any herbs/supplements. Which of the following measures should be implemented for the management of this patient’s condition?
A 50-year-old man presents to your office with fatigue and weakness. He first noticed it a few weeks ago while trying to hang pictures with his wife. His legs have begun to ache as he walks up stairs. He has lost about 20 lbs in the last 3 months. Most recently, he has found that he is more constipated and has trouble rising from the commode. Your physical examination reveals modest proximal weakness, no articular swelling, rash, or any other pertinent findings. Blood work from a recent insurance examination revealed:
Sodium 142 meq/L; potassium 3.8 meq/L; chloride 107 meq/L; bicarbonate 29 meq/L; BUN 30 mg/dL; Cr 1.6 mg/dL; WBC 6.8; Hgb 13.6 g/dL; HCT 40%; MCV 88.0 μm3; platelets 240,000/mm3; AST 200 U/L; ALT 250 U/L; alkaline phosphatase 70 U/L; bilirubin 0.3 mg/dL; ESR 40 mm/h. His CPK is 2400 and an EMG shows fibrillation potentials, positive sleep waves, and myotonic discharges. In addition to addressing his myositis, diagnostic testing should be performed to evaluate for the possibility of which of these?
A 32-year-old man who is HIV-positive was found to have Burkitt’s lymphoma with diffuse bulky abdominal disease. He now reports to the hospital and is scheduled to begin chemotherapy. Admission laboratory studies show elevations of his uric acid at 15 mg/dL, serum phosphorus at 8.5 mg/dL, creatinine at 2.9 mg/dL, and potassium at 6.1 mEq/L. What therapy is most likely to reverse the patient’s metabolic abnormalities?
A 23-year-old African-American presents with acute-onset pain in the abdomen, back, and legs. On physical examination, his pulse is 115 bpm, respiratory rate is 20, blood pressure is 100/70 mmHg, and temperature is 101°F. There is scleral icterus, a systolic ejection murmur at the right upper sternal border, bilateral rhonchi, a right upper quadrant abdominal scar from a cholecystectomy, and a diffusely tender abdomen without rebound. A neurologic examination is normal. A peripheral blood smear is shown in Figure.
Oxygen and IV fluids are given. A urine specific gravity is 1.010. Which of the following is the best explanation for this?
A 45-year-old male comes to your office for his first annual checkup in the last 10 years. On first impression, he appears overweight but is otherwise healthy and has no specific complaints. He has a brother with diabetes and a sister with high blood pressure. Both of his parents are deceased and his father died of a stroke at age 73. He is a long-standing heavy smoker and only drinks alcohol on special
occasions. On physical examination, his blood pressure is 166/90 in the left arm and 164/88 in the right arm. The rest of the examination is unremarkable. He is concerned about his health and does not want to end up on medication, like his siblings. In the initial evaluation of a patient such as this, which of the following should be routinely recommended?
A 35-year-old woman with a history of major depressive disorder is brought into the ED by her boyfriend. He believes she may have overdosed on pain medication in an effort to hurt herself. He gives you three medication bottles which he discovered empty and states that they were nearly full before leaving for work earlier that morning. However, the timing of the ingestion is unclear. All three of the bottles apparently held acetaminophen-containing medications. Examination of the patient reveals a tired-appearing woman complaining of nausea and right upper quadrant abdominal pain. Which of the following pharmacologic treatments is most appropriate at this time?
A 65-year-old man presents to your office for evaluation of abdominal pain. The patient states that he has epigastric pain that radiates to his back. The pain is worse with eating and improves with fasting. The pain has been present for 6 months and is gradually worsening. The patient has lost 15 lbs but feels his oral
intake has been adequate. He complains of greasy stools and frequent thirst and urination. Examination reveals a thin male with temporal wasting and moderate abdominal pain with palpation. The patient consumes approximately 10–15 beers per day and smokes a pack of cigarettes per day for the past 20 years. On further evaluation, the patient is found to be diabetic. He has an elevated HgbA1C and
fasting hyperglycemia. The patient is sent for diabetic teaching sessions and begun on insulin therapy, but is unable to achieve euglycemia. He experiences frequent bouts of symptomatic hypoglycemia requiring ER visits. What is the most likely cause for these episodes?
A 23-year-old female graduate student with acne and asthma presents to you with a chief complaint of headaches. She has noted a gradual increase in the intensity and frequency of the headaches to the point where they are interfering with her daily activities and studies. Your examination shows an obese young lady with papilledema. The remainder of your physical examination is normal. The test ordered above was negative. Which of the following is your most appropriate next
step?
A 72-year-old man comes to your clinic for the first time, accompanied by his wife. His wife states that she is concerned because he has been growing increasingly forgetful over the past year. Within the past month, he has forgotten to turn off the stove and has got lost while walking to the post office one block away from their home. His past medical history is significant for well-controlled diabetes and
chronic lower back pain. He has no history of falls or traumatic injury to the head. Examination of the patient is significant for a score of 18 on a Mini-Mental Status Examination (MMSE). During the administration of the MMSE, the patient blurts out that his wife brought him to the doctor because she is having an extramarital relationship. Use of which of the following medications would be the most likely to lead to worsening of symptoms in this patient?
A 53-year-old White female, with a history of systemic lupus erythematosus (SLE), hypertension, and peripheral vascular disease, is admitted to the hospital for chest pain and dyspnea. Her cardiac enzymes were positive for acute MI. She subsequently undergoes a cardiac catheterization and stenting of the right coronary artery. Her postcardiac catheterization course is unremarkable, and she is discharged home 3 days later with adequate blood pressure control. Five days later, she is brought to the ER by her husband for abdominal pain and nausea. Her medications consist of aspirin, metoprolol, and prednisone. On physical examination, her blood pressure is 190/95 and her heart rate is 85 bpm. In general, she appears nauseated but is in no acute distress. Her cardiac examination reveals a regular rate and rhythm without murmur or rub. Her lung fields are clear bilaterally. The abdominal examination is positive for diffuse discomfort, without guarding or rebound, and normoactive bowel sounds; her stool is positive for occult blood. Her lower extremities have trace edema bilaterally with 2+ distal pulses; moreover, she has a reddish-blue discoloration on both her lower extremities. You retrieve her records from prior hospitalization. The
patient’s laboratory results are as follows:
Which of the following is the optimal therapeutic agent for this patient’s pain management?
A 22-year-old male presents to an acute care clinic in order to have two genital lesions evaluated. He first noticed the lesions about 2 weeks ago, but delayed
seeking medical care because he believed they were harmless due to the lack of any discomfort. He states that he does engage in unprotected sexual intercourse, with the most recent time being 1 month ago. On examination, the glans penis features two distinct nontender papules with elevated edges surrounding ulcerated craters. They each measure 1 cm in diameter. There is also nontender bilateral inguinal lymphadenopathy. Six hours after treating this patient, he calls
your office with complaints of new-onset headache, myalgia, and malaise. He also states that he felt feverish immediately prior to calling and measured his temperature, which was 99.8°F. Which of the following is most appropriate at this time?
A 30-year-old female presents to your office for the evaluation of a rash on her back. It has been present and growing for about a week. Along with this rash, she has had a fever, headache, myalgias, and fatigue. Her symptoms started about a week after returning from a camping trip to New England. She denies having any
bites from ticks or other insects and exposure to poison ivy and has had no wounds to her skin. On examination, her temperature is 99.5°F and her vital signs are otherwise normal. Her rash is shown in Figure. Her examination is otherwise unremarkable.
What is the most likely cause of her rash?
A 64 year old woman presents with bilateral symmetric arthralgias and morning stiffness for several years. She says that she has been worked up for RA in the past. On review of her records as well as the examination you note subcutaneous nodules, positive rheumatoid factor, and radiographs of the hands that revealed joint erosions. Which of her findings has the highest positive likelihood ratio (LR) for the diagnosis of RA?
A 28-year-old woman presents for evaluation of primary infertility. She has had fewer than four periods per year since menarche at age 14, facial hirsutism, acne, and weight gain. On examination, she has a BP 150/100. Her body mass index (BMI) is 40. Acanthosis nigricans is noted along the posterior surface of her neck. Which of the following would be her most likely fasting lipid profile?
A 20-year-old male has had a recent wide local excision of a 1.5 mm melanoma from the right ankle. There is no evidence of metastatic disease. Which of the following is true regarding melanoma?
A 74-year-old male with gout, osteoporosis, and type II diabetes presents for routine follow-up. As you review his medication list you note that he is on insulin, vitamin D, glypizide, quinidine, and allopurinol. You now diagnose him with hypertension that requires pharmacologic management. Which of the following
medications would be contraindicated in this patient?
An 18-year-old female presents for follow-up of a rash. She states that she has been using OTC antifungal preparations without success. She used OTC cortisone cream with mild improvement, but the lesions never disappeared. Her condition also recurred with full severity once the cortisone cream was discontinued. Her family history reveals a first degree relative with similar rash. The rash is over the elbows, trunk, and scalp (see Figure).
Which of the following would be the appropriate initial treatment?
A 45-year-old female develops fever, dysuria, and back pain and is admitted to the hospital after evaluation in the ER discloses pyelonephritis. The patient is placed on broad-spectrum antibiotics and has a good improvement in her symptoms. On hospital day 4, the patient develops a new fever, leukocytosis, and profuse
watery diarrhea. A colonoscopy is performed and the following finding is seen (see Figure):
What is the first-line therapy for treating this disorder?
A 35-year-old woman presents to your office complaining of fatigue and global achiness. She states that she has “not been myself” since she developed a bad whiplash after a motor vehicle accident. Her health has otherwise been good. About 3 years ago, she saw a cardiologist for chest pain. A full evaluation ensued including heart catheterization that showed no coronary disease, although her cholesterol levels were elevated and a statin was prescribed. She sleeps poorly and notes that she has gained a considerable amount of weight. She has seen a gastroenterologist who has told her that her abdominal pain and alternating constipation and diarrhea are because of irritable bowel syndrome. Physical examination shows that her height is 5 ft 2 in. and her weight is 240 lb. Blood pressure is 126/78. Pulse is 86 and regular. Heart and lung examinations are completely normal. Her pharynx is normal and she has no lymphadenopathy. Abdominal examination shows diffuse mild tenderness, but no masses, rebound, guarding, or organomegaly. Rectal and pelvic examinations are normal. Muscular strength is 4/5 distally and proximally, but there is a considerable give way secondary to pain. She is tender bilaterally at the occiput across the trapezius, iliac crest at the greater trochanteric, anserine bursae bilaterally, and at the second intercostal space bilaterally. Reasonable initial evaluations would include
which of the following?
A 72-year-old previously healthy woman was diagnosed with Stage II breast cancer and was initiated on FAC chemotherapy (5-fluorouracil, doxorubicin, cyclophosphamide) 3 months ago. She now presents to the emergency room complaining of exertional dyspnea, orthopnea, and lower extremity edema. Her vital signs are normal, her EKG is normal, and her chest x-ray shows cardiomegaly and pulmonary vascular congestion. What is the most likely cause of the patient’s new symptoms?
A 23-year-old African-American presents with acute-onset pain in the abdomen, back, and legs. On physical examination, his pulse is 115 bpm, respiratory rate is 20, blood pressure is 100/70 mmHg, and temperature is 101°F. There is scleral icterus, a systolic ejection murmur at the right upper sternal border, bilateral rhonchi, a right upper quadrant abdominal scar from a cholecystectomy, and a diffusely tender abdomen without rebound. A neurologic examination is normal. A peripheral blood smear is shown in Figure.
The patient develops pain in both hips and an MRI shows avascular necrosis. This is likely due to which of the following?
A 45-year-old male comes to your office for his first annual checkup in the last 10 years. On first impression, he appears overweight but is otherwise healthy and has no specific complaints. He has a brother with diabetes and a sister with high blood pressure. Both of his parents are deceased and his father died of a stroke at age 73. He is a long-standing heavy smoker and only drinks alcohol on special
occasions. On physical examination, his blood pressure is 166/90 in the left arm and 164/88 in the right arm. The rest of the examination is unremarkable. He is concerned about his health and does not want to end up on medication, like his siblings. Which of the following describes the patient’s blood pressure status?
A 35-year-old woman with a history of major depressive disorder is brought into the ED by her boyfriend. He believes she may have overdosed on pain medication in an effort to hurt herself. He gives you three medication bottles which he discovered empty and states that they were nearly full before leaving for work earlier that morning. However, the timing of the ingestion is unclear. All three of the bottles apparently held acetaminophen-containing medications. Examination of the patient reveals a tired-appearing woman complaining of nausea and right upper quadrant abdominal pain. Which of the following is the worst prognostic indicator if present in this patient?
A 65-year-old man presents to your office for evaluation of abdominal pain. The patient states that he has epigastric pain that radiates to his back. The pain is worse with eating and improves with fasting. The pain has been present for 6 months and is gradually worsening. The patient has lost 15 lbs but feels his oral
intake has been adequate. He complains of greasy stools and frequent thirst and urination. Examination reveals a thin male with temporal wasting and moderate abdominal pain with palpation. The patient consumes approximately 10–15 beers per day and smokes a pack of cigarettes per day for the past 20 years. The patient’s weight loss would be best treated by which of the following regimens?
A 23-year-old female graduate student with acne and asthma presents to you with a chief complaint of headaches. She has noted a gradual increase in the intensity and frequency of the headaches to the point where they are interfering with her daily activities and studies. Your examination shows an obese young lady with papilledema. The remainder of your physical examination is normal. Which of the following is most commonly associated with this condition?
A 72-year-old man comes to your clinic for the first time, accompanied by his wife. His wife states that she is concerned because he has been growing increasingly forgetful over the past year. Within the past month, he has forgotten to turn off the stove and has got lost while walking to the post office one block away from their home. His past medical history is significant for well-controlled diabetes and
chronic lower back pain. He has no history of falls or traumatic injury to the head. Examination of the patient is significant for a score of 18 on a Mini-Mental Status Examination (MMSE). During the administration of the MMSE, the patient blurts out that his wife brought him to the doctor because she is having an extramarital relationship. Despite appropriate treatment, the patient experiences a gradual decline in mental function. He develops erratic sleep habits, frequently awakening at night and wandering throughout his home. His wife states that she once found him sitting on the ground in their yard, unable to recall how he arrived there.
During your latest examination of the patient, you note that he has lost the ability to sign his name, holding the pen as if he is unsure of what to do with it. Exasperated, his wife states that he is now dependent on her for performance of his activities of daily living. Which of the following is indicated in the treatment of
the patient’s condition at its current severity?
A 53-year-old White female, with a history of systemic lupus erythematosus (SLE), hypertension, and peripheral vascular disease, is admitted to the hospital for chest pain and dyspnea. Her cardiac enzymes were positive for acute MI. She subsequently undergoes a cardiac catheterization and stenting of the right coronary artery. Her postcardiac catheterization course is unremarkable, and she is discharged home 3 days later with adequate blood pressure control. Five days later, she is brought to the ER by her husband for abdominal pain and nausea. Her medications consist of aspirin, metoprolol, and prednisone. On physical examination, her blood pressure is 190/95 and her heart rate is 85 bpm. In general, she appears nauseated but is in no acute distress. Her cardiac examination reveals a regular rate and rhythm without murmur or rub. Her lung fields are clear bilaterally. The abdominal examination is positive for diffuse discomfort, without guarding or rebound, and normoactive bowel sounds; her stool is positive for occult blood. Her lower extremities have trace edema bilaterally with 2+ distal pulses; moreover, she has a reddish-blue discoloration on both her lower extremities. You retrieve her records from prior hospitalization. The
patient’s laboratory results are as follows:
Which of the following laboratory findings would be most suggestive of active lupus nephritis?
A 26-year-old HIV-positive man is admitted to the hospital for treatment of a varicella-zoster infection. On the fourth day of treatment, he develops an acute renal insufficiency. What is the most likely treatment-related mechanism accounting for the patient’s acute renal insufficiency?
A 53-year-old fisherman develops pain and swelling of the right hand 8 hours after suffering a fish hook injury to the finger. On physical examination, the patient’s temperature is 102.8°F and the patient appears septic. The patient’s hand and a Gram stain of material aspirated from a bulla are shown in Figures (a) and (b).
(a)
(b)
After appropriate wound care and debridement of necrotic tissue as necessary, which antibiotics should be started in this patient?
A 58-year-old woman is concerned about her risk for osteoporosis and is seen by her general internist. Her mother was diagnosed with osteoporosis and had a hip fracture at age 84. She has no personal or family history of kidney stones or ulcer disease, and she has never had a fracture. She had a hysterectomy at age 48
and took estradiol for 2 years, but discontinued because of a fear of adverse effects. She does not have any vasomotor symptoms. She takes 1500 mg of calcium carbonate and 400 IU vitamin D daily. She is not on any other medications. On examination, she appears well developed and there is no evidence of kyphosis. ABMD test is performed that demonstrates a T score in the spine of ––3.5 and in the hip of –2.8. CXR and mammogram are normal.
Further evaluation demonstrates the following:
Which of the following is the most likely diagnosis?
A 28-year-old woman presents for evaluation of primary infertility. She has had fewer than four periods per year since menarche at age 14, facial hirsutism, acne, and weight gain. On examination, she has a BP 150/100. Her body mass index (BMI) is 40. Acanthosis nigricans is noted along the posterior surface of her neck. How would you counsel this patient?
A 43-year-old patient presents with his fourth episode of culture-proven shingles in a T7 distribution. What is the most likely associated underlying condition?
A 60-year-old male with a history of hypertension and hyperlipidemia undergoes an evaluation for angina. He states that he routinely experiences dyspnea, fatigue, and retrosternal chest discomfort when performing activities such as walking around the block on which his house is located or climbing the flight of stairs within his home. Besides taking medications for his blood pressure and cholesterol, he uses nitroglycerin which successfully alleviates his symptoms. Which of the following best describes the severity of this patient’s angina?
A 60-year-old morbidly obese man presents with complaints of fatigue, worsening exertional dyspnea, three-pillow orthopnea, lower extremity edema, and cough occasionally productive of frothy sputum. He has a long-standing history of type II
diabetes and hypertension. On examination, you note the presence of bibasilar rales, an S3 gallop, jugular venous distention, and 2+ pitting edema in both legs up to the knees. There does not appear to be an arrhythmia present. Which test would be most sensitive for diagnosis of this condition?
A 45-year-old male presents to the hospital for acute abdominal pain and is found to have acute pancreatitis. He has no past medical history but recently has noticed urinary frequency and muscle weakness. He takes no medications. He denies alcohol use. His liver function tests during the episode are normal and magnetic resonance cholangiopancreatography study (MRCP) demonstrates an absence of stones in the biliary tree as well as a normal pancreatic duct. His serum calcium is found to be markedly elevated during this episode. The patient recovers clinically, and repeat serum calcium is also found to be elevated 1 month
after hospital discharge. What is the most likely cause of his hypercalcemia?
A 35-year-old woman presents to your office complaining of fatigue and global achiness. She states that she has “not been myself” since she developed a bad whiplash after a motor vehicle accident. Her health has otherwise been good. About 3 years ago, she saw a cardiologist for chest pain. A full evaluation ensued including heart catheterization that showed no coronary disease, although her cholesterol levels were elevated and a statin was prescribed. She sleeps poorly and notes that she has gained a considerable amount of weight. She has seen a gastroenterologist who has told her that her abdominal pain and alternating constipation and diarrhea are because of irritable bowel syndrome. Physical examination shows that her height is 5 ft 2 in. and her weight is 240 lb. Blood pressure is 126/78. Pulse is 86 and regular. Heart and lung examinations are completely normal. Her pharynx is normal and she has no lymphadenopathy. Abdominal examination shows diffuse mild tenderness, but no masses, rebound, guarding, or organomegaly. Rectal and pelvic examinations are normal. Muscular strength is 4/5 distally and proximally, but there is a considerable give way secondary to pain. She is tender bilaterally at the occiput across the trapezius, iliac crest at the greater trochanteric, anserine bursae bilaterally, and at the second intercostal space bilaterally. In this patient, which of the following conditions may also be exacerbating her symptoms?
A previously healthy 34-year-old man, a lifelong nonsmoker, sought medical care at an Urgent Care Center for an upper respiratory infection. A chest x-ray was obtained, which revealed a peripherally located right lower lobe lung nodule. A follow-up CT of the chest showed the 1.8 cm nodule with multiple nonspecific calcifications, and no associated hilar or mediastinal adenopathy. What is the most appropriate next step?
A 45-year-old male comes to your office for his first annual checkup in the last 10 years. On first impression, he appears overweight but is otherwise healthy and has no specific complaints. He has a brother with diabetes and a sister with high blood pressure. Both of his parents are deceased and his father died of a stroke at age 73. He is a long-standing heavy smoker and only drinks alcohol on special
occasions. On physical examination, his blood pressure is 166/90 in the left arm and 164/88 in the right arm. The rest of the examination is unremarkable. He is concerned about his health and does not want to end up on medication, like his siblings. Your patient returns to clinic a few weeks later for a follow-up appointment. Despite having lost 3 lbs and increasing his activity to walking
2 mi three times per week, his blood pressure remains elevated at 162/92. His initial evaluation revealed a fasting blood sugar of 156 and a hemoglobin (Hgb) A1C of 7.5. Along with starting hypoglycemic medications to control his diabetes, you recommend that he take an antihypertensive medication. At this point, you
decide to start the patient on hypoglycemic medications to control his diabetes. What other intervention is appropriate at this time?
A 72-year-old male with type II diabetes, hypertension, and a history of recurrent pneumonia is admitted to the Medical intensive care unit (ICU) with a diagnosis of septic shock. His vital signs are: BP 80/60 mmHg, RR 24 breaths per minute, pulse 120 beats per minute (bpm), temp. 102.4°F, O2 saturation 99% on room air. Of the choices listed below, what would be your initial management?
A 65-year-old man presents to your office for evaluation of abdominal pain. The patient states that he has epigastric pain that radiates to his back. The pain is worse with eating and improves with fasting. The pain has been present for 6 months and is gradually worsening. The patient has lost 15 lbs but feels his oral
intake has been adequate. He complains of greasy stools and frequent thirst and urination. Examination reveals a thin male with temporal wasting and moderate abdominal pain with palpation. The patient consumes approximately 10–15 beers per day and smokes a pack of cigarettes per day for the past 20 years. The patient’s abdominal pain worsens and his weight loss progresses despite therapy, and you suspect that he may have a malignancy. If a malignancy was present, which tumor marker would be most likely to be elevated in this patient?
A 23-year-old female graduate student with acne and asthma presents to you with a chief complaint of headaches. She has noted a gradual increase in the intensity and frequency of the headaches to the point where they are interfering with her daily activities and studies. Your examination shows an obese young lady with papilledema. The remainder of your physical examination is normal. Which of the following interventions is most appropriate initially for the patient’s suspected diagnosis?
A 28-year-old woman presents to your clinic complaining of feeling “on edge.” Upon further questioning, you discover that she has also noticed problems with irritability, insomnia, fatigue, and restlessness. She also has a history of worrying about things that seem to not bother those around her. She states these symptoms have been present for years but have recently become worse. When you try to gather more information, she interrupts to say that she cannot stay much longer because she is afraid that she will lose her new job as a machinist. Which of the following medications would be most appropriate in this patient?
A 63-year-old Native American male, with a 6-year history of DM, hypertension, and hyperlipidemia, comes to your office as a new patient for a routine examination. He has been experiencing frequent lower back pain and headaches for which he is taking ibuprofen daily for the past 5 weeks. Moreover, he is complaining of mild fatigue. In addition, he is taking aspirin, atorvastatin, verapamil, and glipizide. His physical examination shows a blood pressure of 165/80 and heart rate of 90 bpm. In general, he was not in any distress. His funduscopic examination reveals no signs of diabetic retinopathy. Cardiac examination reveals a regular rate and rhythm with an S4 gallop. His lungs are clear and abdominal examination is unremarkable without any bruit auscultated. He also has 2+ lower extremity pitting edema. Rectal examination reveals brown
stool, negative for occult blood. His laboratory results are as follows:
Which of the following is a typical finding in this patient’s condition?
A 34-year-old amateur spelunker develops cough, dyspnea, and fever 2 weeks after a caving expedition to caves in Kentucky. On physical examination, the
patient’s temperature is 102°F and respiratory rate is 24. On pulmonary examination, there are diffuse crackles bilaterally. A CXR is shown in Figure.
Which of the following is the most likely cause of disease in this patient?
You receive a call from the nurse at a nursing home for a 70-year-old patient of yours who was febrile overnight and had blood cultures, CXR, and urinalysis ordered by the housestaff. The patient was started empirically on a fluoroquinolone orally. The nurse informs you that the CXR and urinalysis were normal but the blood culture grew out Enterococcus faecalis. The patient has been on oral fluoroquinolone for 36 hours and patient is still febrile but appears stable. Which of the following is most appropriate?
Vitamin D supplementation can be helpful in treating which disease?
A 34-year-old female sex worker presents with a several week history of fatigue, malaise, fever, and a 10-lb weight loss. Over the last 2 weeks, the patient noted a rash on her face, torso, arms, legs, palms, and soles (Figures a & b). The patient is HIV negative on a test 2 months ago, has had hepatitis B, gonorrhea, and chlamydia. The patient has an oral temperature of 100.6°F, and generalized lymphadenopathy. The patient does not have any lesions in the mucous membranes.
(a)
(b)
What is the diagnostic test most likely to explain this clinical presentation?
A 24-year-old female presents to your office for excision of a nevus. After obtaining consent and prepping the site, you anesthetize the area with 1% lidocaine. However, as you start the procedure, you note that the patient is not sufficiently anesthetized. Your partner suggests the use of lidocaine with epinephrine. The addition of epinephrine with local anesthetics is useful because of which of the following properties?
A 60-year-old male with a history of hypertension and hyperlipidemia undergoes an evaluation for angina. He states that he routinely experiences dyspnea, fatigue, and retrosternal chest discomfort when performing activities such as walking around the block on which his house is located or climbing the flight of stairs within his home. Besides taking medications for his blood pressure and cholesterol, he uses nitroglycerin which successfully alleviates his symptoms. The patient states that shortly after self-administering nitroglycerin, his heart feels like it races. He does not notice this sensation at any other times. Which of the following interventions would be most appropriate for counteracting this phenomenon?
A 60-year-old morbidly obese man presents with complaints of fatigue, worsening exertional dyspnea, three-pillow orthopnea, lower extremity edema, and cough occasionally productive of frothy sputum. He has a long-standing history of type II
diabetes and hypertension. On examination, you note the presence of bibasilar rales, an S3 gallop, jugular venous distention, and 2+ pitting edema in both legs up to the knees. There does not appear to be an arrhythmia present. Which of the following medications should be given initially?
A 25-year-old woman presents to your office complaining of cold hands. She describes them turning white as she reaches for orange juice in the frozen food section of the supermarket. It seems to be getting worse lately. She has no other symptoms but does note that she and her husband are contemplating pregnancy. Her examination today is unremarkable. What condition is she describing?
An 82-year-old woman schedules an appointment to see you for neck and back pain. At age 50, she had an L4-L5 diskectomy and laminectomy. She also has long-standing hypothyroidism for which she takes levothyroxine 0.1 mg daily. Over the past few months, she has become more fatigued and describes pain in both of her arms, her low back, and the front of her thighs. She notes that the tops of her shoulders are also achy. She decided to call for an appointment because of worsening headache. She tells you that she has an appointment later this afternoon with her ophthalmologist, because she noticed some flickering of the vision in her left eye. Upon further questioning, she does acknowledge that she has cut her telephone conversation short with her daughter because her jaw begins to ache if she talks too long. Physical examination shows that she has normal vital signs. She has diffuse scalp tenderness. The oral mucosa is normal without aphthous ulcers and the salivary pool is normal. Her pupils are equal, round, and reactive to light and accommodation, and extraocular muscles are intact. The funduscopic examination appears normal for her age. Neck motion is
slightly reduced to lateral flexion and rotation. Her trapezii are tender to palpation, but there is no significant loss of range of motion in her shoulders. Her supraspinatus and infraspinatus tendons appear intact. Her quadriceps are mildly tender, but her gastrocnemius muscles are normal. Her strength is normal for age. Her reflexes are normal and symmetrical. The most likely diagnosis is which of the following?
A 32-year-old female presents for her first pap smear in more than 10 years. She has a history of heavy alcohol use and IV drug use and has performed sexual acts for drugs on numerous occasions. Testing performed today reveals her to have chlamydia cervicitis and trichomonas vaginalis and to be seropositive for hepatitis B and hepatitis C. HIV testing is negative. Her pap smear subsequently returns with carcinoma in-situ of the cervix. Infection with which of the following agents is most likely to have resulted in her cancer?
A 52-year-old male construction worker is seen in the emergency department (ED) at a local hospital with complaints of persistent cough for the past 4 months. He has been relatively healthy until a few months ago, when he lost his mother and developed severe depression which left him socially and professionally paralyzed. He has stopped doing any exercise or outdoor activity and spends most of his
time at home eating, sleeping, and watching TV. In addition, he has noticed a 20-lb weight gain over this period but attributes it to his lack of exercise and increased food intake. His cough is worse at night, or any time when he lies down to sleep, and he notices a burning sensation in his throat associated with it. It is not associated with fever or chills, and his wife complains that he is constantly clearing his throat after meals. He smoked a few cigarettes per day as a young man in the Navy but quit more than 30 years ago. He denies recent travel or incarceration and has no recollection of any sick contacts. On examination, he is afebrile and appears mildly obese. His lung examination is clear. His oropharynx is red and mucosal membranes are dry and not inflamed. Which of the following statements describes the likely cause of his chronic cough?
A 42-year-old man without prior significant medical history comes to your office for evaluation of chronic diarrhea of 12 months duration, although the patient states he has had loose stools for many years. During this time he has lost 25 lbs. The diarrhea is large volume, occasionally greasy, and non-bloody. In addition, the patient has mild abdominal pain for much of the day. He has been smoking a pack of cigarettes a day for 20 years and drinks approximately five beers per day. His physical examination reveals a thin male with temporal wasting and generalized muscle loss. He has glossitis and angular cheilosis. He has excoriations on his elbows and knees and scattered papulovesicular lesions in these regions as well. Which of the following is the most likely diagnosis for this patient?
A 60-year-old woman arrives at your office for a routine physical examination. During the course of her examination she asks you about osteoporosis. She is concerned about her risk for osteoporosis, as her mother suffered from multiple vertebral compression fractures at the age of 60. Your patient reports that she still smokes cigarettes (“although I know they are bad for me”) and has one alcoholic beverage a week. She reports having had menopause 5 years ago and experiencing a deep venous thrombosis approximately 20 years ago. She is proud of the fact that she regularly exercises at the local fitness center. She has been taking 1500 mg of calcium with 800 IU of vitamin D every day. You suspect that she is at risk for osteoporosis. Which of the following tests is best to detect
and monitor osteoporosis?
A 64-year-old Hispanic female with type II DM and hypertension for 15 years comes to your office after not seeing a physician for 5 years. The HgbA1C is 9. She reports that her vision has been deteriorating but new glasses from the optometrist have helped. Which of the following findings during your examination would represent the highest risk for blindness in this patient?
A 54-year-old male with uncontrolled type II diabetes and well-controlled hypertension presents with complaints of erectile dysfunction. The patient requests Viagra (sildenafil), as his friends have used it with success.
However, he is concerned as he was told by someone that Viagra can be fatal if used with some blood pressure medications. You would advise the patient that the use of which of the following is contraindicated in patients taking sildenafil?
A 63-year-old Native American male, with a 6-year history of DM, hypertension, and hyperlipidemia, comes to your office as a new patient for a routine examination. He has been experiencing frequent lower back pain and headaches for which he is taking ibuprofen daily for the past 5 weeks. Moreover, he is complaining of mild fatigue. In addition, he is taking aspirin, atorvastatin, verapamil, and glipizide. His physical examination shows a blood pressure of 165/80 and heart rate of 90 bpm. In general, he was not in any distress. His funduscopic examination reveals no signs of diabetic retinopathy. Cardiac examination reveals a regular rate and rhythm with an S4 gallop. His lungs are clear and abdominal examination is unremarkable without any bruit auscultated. He also has 2+ lower extremity pitting edema. Rectal examination reveals brown
stool, negative for occult blood. His laboratory results are as follows:
With regard the workup of this man’s proteinuria, what diagnostic test would you perform next?
A 34-year-old amateur spelunker develops cough, dyspnea, and fever 2 weeks after a caving expedition to caves in Kentucky. On physical examination, the
patient’s temperature is 102°F and respiratory rate is 24. On pulmonary examination, there are diffuse crackles bilaterally. A CXR is shown in Figure.
What diagnostic test would be most appropriate?
A30-year-old female presents to your office for the evaluation of a rash on her back. It has been present and growing for about a week. Along with this rash, she has had a fever, headache, myalgias, and fatigue. Her symptoms started about a week after returning from a camping trip to New England. She denies having any
bites from ticks or other insects and exposure to poison ivy and has had no wounds to her skin. On examination, her temperature is 99.5°F and her vital signs are otherwise normal. Her rash is shown in Figure. Her examination is otherwise unremarkable.
You order IgM and IgG ELISA testing for Borrelia burgdorferi and the results return as negative. Which of the following management options would be most appropriate?
A 34-year-old woman was found to have a 2-cm right thyroid nodule at the time of a well woman examination. The remainder of the thyroid was palpably normal and there were no lymph nodes palpable. There was no history of thyroid disease or radiation therapy to her head or neck. She was clinically euthyroid. Thyroid-stimulating hormone (TSH) was normal. Which of the following tests would be
the most useful in establishing a specific diagnosis?
A 34-year-old female sex worker presents with a several week history of fatigue, malaise, fever, and a 10-lb weight loss. Over the last 2 weeks, the patient noted a rash on her face, torso, arms, legs, palms, and soles (Figures a & b). The patient is HIV negative on a test 2 months ago, has had hepatitis B, gonorrhea, and chlamydia. The patient has an oral temperature of 100.6°F, and generalized lymphadenopathy. The patient does not have any lesions in the mucous membranes.
(a)
(b)
What is the appropriate treatment?
A 64-year-old man with hypertension presents for routine follow-up of his blood pressure. His home blood pressure log reveals readings in the 150/70 range. His home monitor had previously been verified by clinic BP readings. He denies any complaints. His current medications include HCTZ 25 mg daily, metoprolol
100 mg twice daily, enalapril 20 mg twice daily, and amlodipine 10 mg daily. He states he is adherent to his medication, drug, and exercise regimen as you recommended. At this time, how would you advise the patient?
A 60-year-old male with a history of hypertension and hyperlipidemia undergoes an evaluation for angina. He states that he routinely experiences dyspnea, fatigue, and retrosternal chest discomfort when performing activities such as walking around the block on which his house is located or climbing the flight of stairs within his home. Besides taking medications for his blood pressure and cholesterol, he uses nitroglycerin which successfully alleviates his symptoms. The patient undergoes a cardiac catheterization and is found to have 70% narrowing of the left anterior descending and proximal left circumflex arteries. Which of the following would be the most appropriate management of this condition?
A 60-year-old morbidly obese man presents with complaints of fatigue, worsening exertional dyspnea, three-pillow orthopnea, lower extremity edema, and cough occasionally productive of frothy sputum. He has a long-standing history of type II
diabetes and hypertension. On examination, you note the presence of bibasilar rales, an S3 gallop, jugular venous distention, and 2+ pitting edema in both legs up to the knees. There does not appear to be an arrhythmia present. Which of the following has been shown to prolong survival in patients with this condition?
A 48-year-old woman presents for evaluation of progressively worsening dyspnea. She relates the onset of symptoms to a “walking pneumonia” that she had a year ago. Her breathing has worsened progressively since that time. She has a “smoker’s cough” productive of some clear or white phlegm, for which she frequently sucks on cough drops. She started smoking regularly at the age of 18. She currently smokes about a pack of cigarettes a day, down from as much as two packs per day. She is not on any medications regularly. She has no history of
heart disease and has always had normal blood pressure. Which of the following physical examination findings are you most likely to find in this patient?
An 82-year-old woman schedules an appointment to see you for neck and back pain. At age 50, she had an L4-L5 diskectomy and laminectomy. She also has long-standing hypothyroidism for which she takes levothyroxine 0.1 mg daily. Over the past few months, she has become more fatigued and describes pain in both of her arms, her low back, and the front of her thighs. She notes that the tops of her shoulders are also achy. She decided to call for an appointment because of worsening headache. She tells you that she has an appointment later this afternoon with her ophthalmologist, because she noticed some flickering of the vision in her left eye. Upon further questioning, she does acknowledge that she has cut her telephone conversation short with her daughter because her jaw begins to ache if she talks too long. Physical examination shows that she has normal vital signs. She has diffuse scalp tenderness. The oral mucosa is normal without aphthous ulcers and the salivary pool is normal. Her pupils are equal, round, and reactive to light and accommodation, and extraocular muscles are intact. The funduscopic examination appears normal for her age. Neck motion is
slightly reduced to lateral flexion and rotation. Her trapezii are tender to palpation, but there is no significant loss of range of motion in her shoulders. Her supraspinatus and infraspinatus tendons appear intact. Her quadriceps are mildly tender, but her gastrocnemius muscles are normal. Her strength is normal for age. Her reflexes are normal and symmetrical. Which of the following should be done next?
A 32-year-old female presents for her first pap smear in more than 10 years. She has a history of heavy alcohol use and IV drug use and has performed sexual acts for drugs on numerous occasions. Testing performed today reveals her to have chlamydia cervicitis and trichomonas vaginalis and to be seropositive for hepatitis B and hepatitis C. HIV testing is negative. Her pap smear subsequently returns with carcinoma in-situ of the cervix. What should be the next step in her work-up?
A 52-year-old male construction worker is seen in the emergency department (ED) at a local hospital with complaints of persistent cough for the past 4 months. He has been relatively healthy until a few months ago, when he lost his mother and developed severe depression which left him socially and professionally paralyzed. He has stopped doing any exercise or outdoor activity and spends most of his time at home eating, sleeping, and watching TV. In addition, he has noticed a 20-lb weight gain over this period but attributes it to his lack of exercise and increased food intake. His cough is worse at night, or any time when he lies down to sleep, and he notices a burning sensation in his throat associated with it. It is not associated with fever or chills, and his wife complains that he is constantly clearing his throat after meals. He smoked a few cigarettes per day as a young man in the Navy but quit more than 30 years ago. He denies recent travel or incarceration and has no recollection of any sick contacts. On examination, he is afebrile and appears mildly obese. His lung examination is clear. His oropharynx is red and mucosal membranes are dry and not inflamed. Which of the following therapies would be most beneficial in alleviating this patient’s symptoms?
A 42-year-old man without prior significant medical history comes to your office for evaluation of chronic diarrhea of 12 months duration, although the patient states he has had loose stools for many years. During this time he has lost 25 lbs. The diarrhea is large volume, occasionally greasy, and non-bloody. In addition, the patient has mild abdominal pain for much of the day. He has been smoking a pack of cigarettes a day for 20 years and drinks approximately five beers per day. His physical examination reveals a thin male with temporal wasting and generalized muscle loss. He has glossitis and angular cheilosis. He has excoriations on his elbows and knees and scattered papulovesicular lesions in these regions as well. Which of the following is the best test to confirm the suspected diagnosis?
A 60-year-old woman arrives at your office for a routine physical examination. During the course of her examination she asks you about osteoporosis. She is concerned about her risk for osteoporosis, as her mother suffered from multiple vertebral compression fractures at the age of 60. Your patient reports that she still smokes cigarettes (“although I know they are bad for me”) and has one alcoholic beverage a week. She reports having had menopause 5 years ago and experiencing a deep venous thrombosis approximately 20 years ago. She is proud of the fact that she regularly exercises at the local fitness center. She has been taking 1500 mg of calcium with 800 IU of vitamin D every day. You suspect that she is at risk for osteoporosis. After performing the appropriate imaging
study, you determine that your patient has osteoporosis. Of the following choices, which is a risk factor most likely contributing to her osteoporosis?
A 64-year-old Hispanic female with type II DM and hypertension for 15 years comes to your office after not seeing a physician for 5 years. The HgbA1C is 9. She reports that her vision has been deteriorating but new glasses from the optometrist have helped. Your examination findings include all of the above. These form which of the following diagnoses?
Bupivacaine is a local anesthetic agent that is much more potent and the duration of action of which is considerably longer than procaine. Possible reasons for this difference include which of the following?
A 63-year-old Native American male, with a 6-year history of DM, hypertension, and hyperlipidemia, comes to your office as a new patient for a routine examination. He has been experiencing frequent lower back pain and headaches for which he is taking ibuprofen daily for the past 5 weeks. Moreover, he is complaining of mild fatigue. In addition, he is taking aspirin, atorvastatin, verapamil, and glipizide. His physical examination shows a blood pressure of 165/80 and heart rate of 90 bpm. In general, he was not in any distress. His funduscopic examination reveals no signs of diabetic retinopathy. Cardiac examination reveals a regular rate and rhythm with an S4 gallop. His lungs are clear and abdominal examination is unremarkable without any bruit auscultated. He also has 2+ lower extremity pitting edema. Rectal examination reveals brown
stool, negative for occult blood. His laboratory results are as follows:
Which additional of the following would best help in the determination of the etiology of this patient’s nephrotic syndrome?
A 34-year-old amateur spelunker develops cough, dyspnea, and fever 2 weeks after a caving expedition to caves in Kentucky. On physical examination, the
patient’s temperature is 102°F and respiratory rate is 24. On pulmonary examination, there are diffuse crackles bilaterally. A CXR is shown in Figure.
Which of these is the most appropriate statement about infection control of this patient if the patient is hospitalized?
A 39-year-old HIV-positive male presents for routine follow-up. He is on highly active antiretroviral therapy. A CD4 count is 250/μL. His vital signs are within normal limits and his examination is normal. Which of the following management options is most appropriate at this time?
A 32-year-old woman presents with complaints of irritability, heat intolerance, hyperdefecation, and frequent palpitations. She has lost 20 lb over the past six months. She has always been in good health and does not take any prescription or OTC medications. She denies any prior history of thyroid disease or exposure to head/neck irradiation, but she states that one of her relatives was diagnosed with a thyroid disorder at roughly the same age. Vital signs are as follows: BP 138/78, HR 112, RR 22, temp. 98.8°F. On examination, her thyroid is diffusely enlarged and smooth. Auscultation of the thyroid reveals a bruit. Her hair is fine in texture, and she has warm velvety skin. She has hyperactive deep tendon reflexes. There is a fine tremor in her outstretched hands. Which of the following is a common finding in this condition?
A 58-year-old male presents to your office for a well-male examination. It has been several years since he last visited a doctor, but he states that he has been in “excellent health.” He denies any history of drinking, smoking, or using illegal drugs. He maintains a diet low in sodium and fat. An avid sports enthusiast, he also spends at least 2 hours per day engaged in some type of outdoor physical
activity. On physical examination, you discover a translucent waxy papule with raised borders on the posterior aspect of his left shoulder. Which of the following is true of this patient’s skin condition?
C1 deficiency has three subcomponents, of which the most common is deficiency of C1q. Most of those patients will have clinical and serologic findings typical of which of these?
A 64-year-old female with no significant medical history presents with vague complaints of progressive generalized muscle weakness and fatigue. She denies any history of trauma or drug use and does not take any prescription, OTC or herbal medications. This is a new complaint and she has not had any prior workup. There is no evidence of trauma and a recent TSH was normal. On examination, you find mild muscle tenderness and atrophy. She has difficulty standing from a chair unless she pushes up with her arms at the same time. Her
neurologic examination is normal. Which of the following tests would be most helpful in confirming your clinical diagnosis?
A 60-year-old morbidly obese man presents with complaints of fatigue, worsening exertional dyspnea, three-pillow orthopnea, lower extremity edema, and cough occasionally productive of frothy sputum. He has a long-standing history of type II
diabetes and hypertension. On examination, you note the presence of bibasilar rales, an S3 gallop, jugular venous distention, and 2+ pitting edema in both legs up to the knees. There does not appear to be an arrhythmia present. A transesophageal echocardiogram (TEE) is performed which reveals a left ventricular ejection fraction (LVEF) of 30%. Which of the following accurately describes this patient and his condition?
A 48-year-old woman presents for evaluation of progressively worsening dyspnea. She relates the onset of symptoms to a “walking pneumonia” that she had a year ago. Her breathing has worsened progressively since that time. She has a “smoker’s cough” productive of some clear or white phlegm, for which she frequently sucks on cough drops. She started smoking regularly at the age of 18. She currently smokes about a pack of cigarettes a day, down from as much as two packs per day. She is not on any medications regularly. She has no history of
heart disease and has always had normal blood pressure. Which of the following is most likely to be found on a CXR?
A 32-year-old man comes to the office for his annual checkup. He is asymptomatic and his physical exam is normal. He reports that his father died of colon cancer at age 46 and his older brother was recently diagnosed with colon cancer at age 37. His paternal aunt was previously diagnosed and treated for endometrial cancer. He is concerned about his family history of malignancy
and wants to discuss cancer screening. What would be the most appropriate recommendation at this time?
A 32-year-old female presents for her first pap smear in more than 10 years. She has a history of heavy alcohol use and IV drug use and has performed sexual acts for drugs on numerous occasions. Testing performed today reveals her to have chlamydia cervicitis and trichomonas vaginalis and to be seropositive for hepatitis B and hepatitis C. HIV testing is negative. Her pap smear subsequently returns with carcinoma in-situ of the cervix. Subsequent work-up confirms the presence of micro-invasive cervical carcinoma [Stage Ia]. What would be the most appropriate treatment?
You are asked to interview a young couple who wish to conceive a child. Their first and only son was born with a rare, autosomal recessive glycogen storage disorder known as Pompe disease. Both parents are healthy and unaffected by this disease, but the father believes that he has heard of a distant cousin who also has this disease. They are concerned about the possibility that their next child will also be born with the affliction. In giving them advice about their chances of having a healthy child, you should:
A 42-year-old man without prior significant medical history comes to your office for evaluation of chronic diarrhea of 12 months duration, although the patient states he has had loose stools for many years. During this time he has lost 25 lbs. The diarrhea is large volume, occasionally greasy, and non-bloody. In addition, the patient has mild abdominal pain for much of the day. He has been smoking a pack of cigarettes a day for 20 years and drinks approximately five beers per day. His physical examination reveals a thin male with temporal wasting and generalized muscle loss. He has glossitis and angular cheilosis. He has excoriations on his elbows and knees and scattered papulovesicular lesions in these regions as well. What is the most serious long-term complication this patient could face?
A 60-year-old woman arrives at your office for a routine physical examination. During the course of her examination she asks you about osteoporosis. She is concerned about her risk for osteoporosis, as her mother suffered from multiple vertebral compression fractures at the age of 60. Your patient reports that she still smokes cigarettes (“although I know they are bad for me”) and has one alcoholic beverage a week. She reports having had menopause 5 years ago and experiencing a deep venous thrombosis approximately 20 years ago. She is proud of the fact that she regularly exercises at the local fitness center. She has been taking 1500 mg of calcium with 800 IU of vitamin D every day. You suspect that she is at risk for osteoporosis. After a thorough discussion with your patient,
you determine that pharmacologic intervention would be beneficial given the severity of her osteoporosis. Which of the following is most appropriate for your patient?
A 54-year-old Asian female with no significant medical history presents with frontal headache, eye pain, nausea, and vomiting. Her abdominal examination shows mild diffuse tenderness but no rebound or guarding. Her mucous membranes are dry. Her vision is blurry in both eyes, her eyes are injected but her extraocular muscles are intact. Her pupils are mid-dilated and fixed. Which of the following is most likely to provide a diagnosis?
A 68-year-old White male, with a history of hypertension, an 80 pack-year history of tobacco use and emphysema, is brought into the ER because of 4 days of progressive confusion and lethargy. His wife notes that he takes amlodipine for his hypertension. He does not use over-the-counter (OTC) medications, alcohol, or drugs. Furthermore, she indicates that he has unintentionally lost approximately 30 lbs in the last 6 months. His physical examination shows that he is afebrile with a blood pressure of 142/85, heart rate of 92 (no orthostatic changes), and a room-air O2 saturation of 91%. He is 70 kg. The patient appears cachectic. He is arousable but lethargic and unable to follow any commands. His mucous membranes are moist, heart rate regular without murmurs or a S3/S4 gallop, and extremities without any edema. His pulmonary examination shows mildly diminished breath sounds in the right lower lobe with wheezing bilaterally. The patient is unable to follow commands during neurologic examination but moves all his extremities spontaneously. Laboratory results are as follows:
What is the most likely cause of this patient’s altered mental status?
A 63-year-old Native American male, with a 6-year history of DM, hypertension, and hyperlipidemia, comes to your office as a new patient for a routine examination. He has been experiencing frequent lower back pain and headaches for which he is taking ibuprofen daily for the past 5 weeks. Moreover, he is complaining of mild fatigue. In addition, he is taking aspirin, atorvastatin, verapamil, and glipizide. His physical examination shows a blood pressure of 165/80 and heart rate of 90 bpm. In general, he was not in any distress. His funduscopic examination reveals no signs of diabetic retinopathy. Cardiac examination reveals a regular rate and rhythm with an S4 gallop. His lungs are clear and abdominal examination is unremarkable without any bruit auscultated. He also has 2+ lower extremity pitting edema. Rectal examination reveals brown
stool, negative for occult blood. His laboratory results are as follows:
Which of the following antihypertensive medications would be best implemented in patients with diabetic nephropathy?
A 53-year-old insulin-dependent diabetic, who underwent a cadaveric renal transplant 1 year prior to admission, presents with fever and cough of 3 weeks duration. He works as a long-haul trucker, carting fruit from McAllen, Texas (on the Texas-Mexico border) to Fresno, California. He does not smoke. His PPD skin test prior to admission was positive. On physical examination, his respiratory
rate is 25, his oral temperature is 101°F, his lungs have rhonchi and decreased breath sounds on the left. His CXR is shown in Figure.
What organism besides Mycobacterium tuberculosis leads your differential as a cause of pneumonia in this case?
A 39-year-old HIV-positive male presents for routine follow-up. He is on highly active antiretroviral therapy. A CD4 count is 250/μL. His vital signs are within normal limits and his examination is normal. He has a PPD placed and follows up in 48 hours. At the site of the injection you find 6 mm of induration. A CXR is normal. He has never been treated for tuberculosis or a positive PPD before. Which management option is most appropriate?
A 32-year-old woman presents with complaints of irritability, heat intolerance, hyperdefecation, and frequent palpitations. She has lost 20 lb over the past six months. She has always been in good health and does not take any prescription or OTC medications. She denies any prior history of thyroid disease or exposure to head/neck irradiation, but she states that one of her relatives was diagnosed with a thyroid disorder at roughly the same age. Vital signs are as follows: BP 138/78, HR 112, RR 22, temp. 98.8°F. On examination, her thyroid is diffusely enlarged and smooth. Auscultation of the thyroid reveals a bruit. Her hair is fine in texture, and she has warm velvety skin. She has hyperactive deep tendon reflexes. There is a fine tremor in her outstretched hands. Which of the following sets of laboratory results would be consistent with this patient’s presentation?
A 58-year-old male presents to your office for a well-male examination. It has been several years since he last visited a doctor, but he states that he has been in “excellent health.” He denies any history of drinking, smoking, or using illegal drugs. He maintains a diet low in sodium and fat. An avid sports enthusiast, he also spends at least 2 hours per day engaged in some type of outdoor physical
activity. On physical examination, you discover a translucent waxy papule with raised borders on the posterior aspect of his left shoulder. Which of the following is the most important risk factor for development of this skin condition?
A21-year-old Asian female, with past medical history of exertional asthma, comes to your office complaining of mild low back pain. It started after her working out in the gym 3 days ago. The pain is 2–4 out of 10 in intensity, has no radiation, increases with bending or lying down for a long time, and improves with warm showers. You examine the patient, diagnose her with paravertebral muscle spasm, and give her prescriptions for cyclobenzaprine and naproxen to use as needed for pain and stiffness. You receive a call from your patient 2 hours
later. She is having generalized itching, dizziness, and swelling of the tongue and lips. She is having difficulty breathing. She tells you that she took the first dose of the medication you prescribed about 30 minutes ago. What should you do at this time?
A 64-year-old female with no significant medical history presents with vague complaints of progressive generalized muscle weakness and fatigue. She denies any history of trauma or drug use and does not take any prescription, OTC or herbal medications. This is a new complaint and she has not had any prior workup. There is no evidence of trauma and a recent TSH was normal. On examination, you find mild muscle tenderness and atrophy. She has difficulty standing from a chair unless she pushes up with her arms at the same time. Her
neurologic examination is normal. Your patient’s test result confirms your clinical
suspicion. The patient’s symptoms have become more severe. Which of the following treatment options would be most appropriate?
A 70-year-old male is seen in the office for chest pain. He reports that he is getting substernal chest pain, without radiation, when he mows his lawn. The pain resolves with 10–15 minutes of rest. He has never had pain at rest. He has no other cardiac complaints and his review of systems is otherwise negative. He has an unremarkable medical history and takes only a baby aspirin a day. On examination, his blood pressure is 160/70, pulse 85, and respiratory rate 16. His cardiac examination is notable for a harsh, 3/6 systolic ejection murmur along the sternal border that radiates to the carotid arteries. His carotid pulsation is noted to rise slowly and is small and sustained. His lungs are clear. The remainder of his examination is normal. Which of the following would be most likely to be seen on an ECG?
A 48-year-old woman presents for evaluation of progressively worsening dyspnea. She relates the onset of symptoms to a “walking pneumonia” that she had a year ago. Her breathing has worsened progressively since that time. She has a “smoker’s cough” productive of some clear or white phlegm, for which she frequently sucks on cough drops. She started smoking regularly at the age of 18. She currently smokes about a pack of cigarettes a day, down from as much as two packs per day. She is not on any medications regularly. She has no history of
heart disease and has always had normal blood pressure. You recommend smoking cessation to your patient. She asks why, at this point, she should quit. Which of the following statements is true?
Which of the following risk factors has the strongest association with the development of malignant melanoma?
Numerous types of cancers are associated with infectious diseases. For which of the following cancers is there a vaccine currently available against the infectious agent which leads to the tumor?
A 53-year-old female has made an appointment to see you concerning the recent onset of menopause. Her last menstrual period was 8 months ago and, over the last year, she had noticed that her periods were becoming lighter and less frequent. In addition, she has developed frequent hot flashes, and her mood has become very labile. She wishes to know what your advice is regarding hormone
replacement therapy (HRT). She has heard recent reports in the news concerning an increased risk of developing cardiovascular complications, especially heart attacks and strokes. Although she is in great health, her father died at age 50 of a massive heart attack. Her mother is alive and well, and there is no history of breast cancer among the females in her family. Regarding postmenopausal HRT, which of the following statements would be correct?
A 24-year-old male medical student is admitted to the hospital for the evaluation of a 3-month history of bloody stools. The patient has approximately six blood stained or blood streaked stools per day, associated with relatively little, if any, pain. He has not had any weight loss, and he has been able to attend classes without interruption. He denies any fecal incontinence. He has no prior medical history. Review of systems is remarkable only for occasional fevers and the fact that the patient quit smoking approximately 8 months ago. A colonoscopy is performed and reveals a granular, friable colonic mucosal surface with loss of normal vascular pattern from the anal verge to the hepatic flexure of the colon. Biopsies reveal prominent neutrophils in the epithelium and cryptitis with focal crypt abscesses, and no dysplasia. The patient is diagnosed with ulcerative colitis. Which of the following is the best initial treatment for this patient?
A 28-year-old male, well known to your clinic, presents for management of swelling, pain, and tenderness that has developed in his left ankle and right
knee. It has persisted for 1 month. Your patient reports that he developed severe diarrhea after a picnic 1 month prior to the onset of his arthritis. During the interval between the diarrhea and onset of arthritis, he developed a “pink eye” that lasted for 4 days. He denies any symptoms of back pain or stiffness. You remember that he was treated with ceftriaxone and doxycycline for gonorrhea 2 years ago, which he acquired from sexual activity with multiple partners. Since that time, he has been in a monogamous relationship with his wife and has not
had any genitourinary symptoms. He promises that he has been faithful to his wife and has not engaged in unprotected sexual activity outside his marriage.
His physical examination is notable for a swollen left ankle, swollen right knee, and the absence of penile discharge or any skin lesions. Which of the following is the most likely diagnosis?
A 54-year-old Asian female with no significant medical history presents with frontal headache, eye pain, nausea, and vomiting. Her abdominal examination shows mild diffuse tenderness but no rebound or guarding. Her mucous membranes are dry. Her vision is blurry in both eyes, her eyes are injected but her extraocular muscles are intact. Her pupils are mid-dilated and fixed. What other finding is this patient most likely to have?
A 68-year-old White male, with a history of hypertension, an 80 pack-year history of tobacco use and emphysema, is brought into the ER because of 4 days of progressive confusion and lethargy. His wife notes that he takes amlodipine for his hypertension. He does not use over-the-counter (OTC) medications, alcohol, or drugs. Furthermore, she indicates that he has unintentionally lost approximately 30 lbs in the last 6 months. His physical examination shows that he is afebrile with a blood pressure of 142/85, heart rate of 92 (no orthostatic changes), and a room-air O2 saturation of 91%. He is 70 kg. The patient appears cachectic. He is arousable but lethargic and unable to follow any commands. His mucous membranes are moist, heart rate regular without murmurs or a S3/S4 gallop, and extremities without any edema. His pulmonary examination shows mildly diminished breath sounds in the right lower lobe with wheezing bilaterally. The patient is unable to follow commands during neurologic examination but moves all his extremities spontaneously. Laboratory results are as follows:
Which of the following provides the best explanation for this patient’s hyponatremia?
A 63-year-old Native American male, with a 6-year history of DM, hypertension, and hyperlipidemia, comes to your office as a new patient for a routine examination. He has been experiencing frequent lower back pain and headaches for which he is taking ibuprofen daily for the past 5 weeks. Moreover, he is complaining of mild fatigue. In addition, he is taking aspirin, atorvastatin, verapamil, and glipizide. His physical examination shows a blood pressure of 165/80 and heart rate of 90 bpm. In general, he was not in any distress. His funduscopic examination reveals no signs of diabetic retinopathy. Cardiac examination reveals a regular rate and rhythm with an S4 gallop. His lungs are clear and abdominal examination is unremarkable without any bruit auscultated. He also has 2+ lower extremity pitting edema. Rectal examination reveals brown
stool, negative for occult blood. His laboratory results are as follows:
Which of the following microscopic findings on kidney biopsy is most usually associated with HIV infection?
A 53-year-old insulin-dependent diabetic, who underwent a cadaveric renal transplant 1 year prior to admission, presents with fever and cough of 3 weeks duration. He works as a long-haul trucker, carting fruit from McAllen, Texas (on the Texas-Mexico border) to Fresno, California. He does not smoke. His PPD skin test prior to admission was positive. On physical examination, his respiratory
rate is 25, his oral temperature is 101°F, his lungs have rhonchi and decreased breath sounds on the left. His CXR is shown in Figure.
What is the best diagnostic approach?
A 39-year-old HIV-positive male presents for routine follow-up. He is on highly active antiretroviral therapy. A CD4 count is 250/μL. His vital signs are within normal limits and his examination is normal. One month later, a repeat measurement of the patient’s CD4 count is 225/μL. Which of the following interventions would be the most appropriate at this time?
A 32-year-old woman presents with complaints of irritability, heat intolerance, hyperdefecation, and frequent palpitations. She has lost 20 lb over the past six months. She has always been in good health and does not take any prescription or OTC medications. She denies any prior history of thyroid disease or exposure to head/neck irradiation, but she states that one of her relatives was diagnosed with a thyroid disorder at roughly the same age. Vital signs are as follows: BP 138/78, HR 112, RR 22, temp. 98.8°F. On examination, her thyroid is diffusely enlarged and smooth. Auscultation of the thyroid reveals a bruit. Her hair is fine in texture, and she has warm velvety skin. She has hyperactive deep tendon reflexes. There is a fine tremor in her outstretched hands. Which of the following interventions is most
appropriate at this time?
A 58-year-old male presents to your office for a well-male examination. It has been several years since he last visited a doctor, but he states that he has been in “excellent health.” He denies any history of drinking, smoking, or using illegal drugs. He maintains a diet low in sodium and fat. An avid sports enthusiast, he also spends at least 2 hours per day engaged in some type of outdoor physical
activity. On physical examination, you discover a translucent waxy papule with raised borders on the posterior aspect of his left shoulder. The treatment modality associated with the lowest recurrence rate is which of these?
A21-year-old Asian female, with past medical history of exertional asthma, comes to your office complaining of mild low back pain. It started after her working out in the gym 3 days ago. The pain is 2–4 out of 10 in intensity, has no radiation, increases with bending or lying down for a long time, and improves with warm showers. You examine the patient, diagnose her with paravertebral muscle spasm, and give her prescriptions for cyclobenzaprine and naproxen to use as needed for pain and stiffness. You receive a call from your patient 2 hours
later. She is having generalized itching, dizziness, and swelling of the tongue and lips. She is having difficulty breathing. She tells you that she took the first dose of the medication you prescribed about 30 minutes ago. The most beneficial immediate intervention for this patient would be which of the following?
A67-year-old female with past medical history of rheumatoid arthritis on chronic steroid treatment and past surgical history (PSH) of complete hysterectomy secondary to fibroids presents for routine visit. Patient states that she has had multiple arthralgias worsening over the last 2 years. She had a DEXA scan done that showed a T score of −1.5. She has been taking calcium + vitamin D,
and even started an exercise program at her local gym. She was started on bisphosphonates, which she has tolerated well. Prior to discharging the patient, how soon would you counsel her to repeat the DEXA scan?
A 70-year-old male is seen in the office for chest pain. He reports that he is getting substernal chest pain, without radiation, when he mows his lawn. The pain resolves with 10–15 minutes of rest. He has never had pain at rest. He has no other cardiac complaints and his review of systems is otherwise negative. He has an unremarkable medical history and takes only a baby aspirin a day. On examination, his blood pressure is 160/70, pulse 85, and respiratory rate 16. His cardiac examination is notable for a harsh, 3/6 systolic ejection murmur along the sternal border that radiates to the carotid arteries. His carotid pulsation is noted to rise slowly and is small and sustained. His lungs are clear. The remainder of his examination is normal. Which of the following would be the most appropriate test to order next?
A 19-year-old woman begins chemotherapy for an acute leukemia. Although you determine that her renal function is unimpaired prior to the initiation of treatment, you feel that she may be at high risk for development of tumor lysis syndrome given her condition’s typically good response to chemotherapy. Which of the following is an appropriate medication to use as a preventative measure prior to and during her treatment for leukemia?
A 54-year-old man without significant past medical history presents to his primary care physician complaining of epigastric discomfort and early satiety. He subsequently undergoes an endoscopic procedure revealing an ulcerated mucosal lesion. The biopsy of this lesion is interpreted as a well-differentiated lymphoma. Which of the following statements regarding his treatment and prognosis is most accurate?
A 24-year-old male presents to the office for evaluation of a nodule on his left testicle. He noticed the mass while washing in the shower. He has had no pain, no weight loss, no change in sexual functioning, and no blood in his semen. Examination reveals the presence of a firm, nontender, 1 cm nodule on the testicle. No other masses and no inguinal adenopathy are noted. Ultrasound of the scrotum confirms that the mass is on the testicle. What would be the next step in management?
A 53-year-old female has made an appointment to see you concerning the recent onset of menopause. Her last menstrual period was 8 months ago and, over the last year, she had noticed that her periods were becoming lighter and less frequent. In addition, she has developed frequent hot flashes, and her mood has become very labile. She wishes to know what your advice is regarding hormone
replacement therapy (HRT). She has heard recent reports in the news concerning an increased risk of developing cardiovascular complications, especially heart attacks and strokes. Although she is in great health, her father died at age 50 of a massive heart attack. Her mother is alive and well, and there is no history of breast cancer among the females in her family. Which of the following would be the strongest argument to avoid HRT in this patient?
A 24-year-old male medical student is admitted to the hospital for the evaluation of a 3-month history of bloody stools. The patient has approximately six blood stained or blood streaked stools per day, associated with relatively little, if any, pain. He has not had any weight loss, and he has been able to attend classes without interruption. He denies any fecal incontinence. He has no prior medical history. Review of systems is remarkable only for occasional fevers and the fact that the patient quit smoking approximately 8 months ago. A colonoscopy is performed and reveals a granular, friable colonic mucosal surface with loss of normal vascular pattern from the anal verge to the hepatic flexure of the colon. Biopsies reveal prominent neutrophils in the epithelium and cryptitis with focal crypt abscesses, and no dysplasia. The patient is diagnosed with ulcerative colitis. While on the inpatient service, the patient is noted to have a serum alkaline phosphatase of 380 U/L and a bilirubin of 2.4 mg/dL. An ERCP is performed, and the following cholangiogram is obtained (see Figure below). In addition to ulcerative colitis, the patient likely has what other illness?
A 28-year-old male, well known to your clinic, presents for management of swelling, pain, and tenderness that has developed in his left ankle and right
knee. It has persisted for 1 month. Your patient reports that he developed severe diarrhea after a picnic 1 month prior to the onset of his arthritis. During the interval between the diarrhea and onset of arthritis, he developed a “pink eye” that lasted for 4 days. He denies any symptoms of back pain or stiffness. You remember that he was treated with ceftriaxone and doxycycline for gonorrhea 2 years ago, which he acquired from sexual activity with multiple partners. Since that time, he has been in a monogamous relationship with his wife and has not
had any genitourinary symptoms. He promises that he has been faithful to his wife and has not engaged in unprotected sexual activity outside his marriage.
His physical examination is notable for a swollen left ankle, swollen right knee, and the absence of penile discharge or any skin lesions. What would be the appropriate management for this patient’s arthritis?
A 54-year-old Asian female with no significant medical history presents with frontal headache, eye pain, nausea, and vomiting. Her abdominal examination shows mild diffuse tenderness but no rebound or guarding. Her mucous membranes are dry. Her vision is blurry in both eyes, her eyes are injected but her extraocular muscles are intact. Her pupils are mid-dilated and fixed. Which of the following is the most likely diagnosis?
A 68-year-old White male, with a history of hypertension, an 80 pack-year history of tobacco use and emphysema, is brought into the ER because of 4 days of progressive confusion and lethargy. His wife notes that he takes amlodipine for his hypertension. He does not use over-the-counter (OTC) medications, alcohol, or drugs. Furthermore, she indicates that he has unintentionally lost approximately 30 lbs in the last 6 months. His physical examination shows that he is afebrile with a blood pressure of 142/85, heart rate of 92 (no orthostatic changes), and a room-air O2 saturation of 91%. He is 70 kg. The patient appears cachectic. He is arousable but lethargic and unable to follow any commands. His mucous membranes are moist, heart rate regular without murmurs or a S3/S4 gallop, and extremities without any edema. His pulmonary examination shows mildly diminished breath sounds in the right lower lobe with wheezing bilaterally. The patient is unable to follow commands during neurologic examination but moves all his extremities spontaneously. Laboratory results are as follows:
Which of the following would be the optimal choice of solution to infuse in order to adequately correct this patient’s hyponatremia?
A patient you see routinely in the clinic has elevated liver function tests. ALT is 89, AST is 75, and the total bilirubin and alkaline phosphatase are normal. The patient has no past history of hepatitis, taking medications, or excessive drinking. You order hepatitis serologies. The results are as follows: Positive: HBsAg and anti-HBc. Negative: anti-HBs, anti-HBc IgM, anti-HAV, and anti-HCV What is your interpretation?
A 76-year-old alcoholic male with hypertension, type II diabetes, and a history of congestive heart presented with cough, fever, malaise, and chills. His initial vitals were: HR 110, T: 102°F, RR: 25, BP 90/60, O2 saturation 93% on 4L/NC. The patient decompensated in the ER and was intubated. Intubation was achieved after three attempts secondary to patient vomiting during the initial attempts. Patient was admitted to the ICU with a diagnosis of sepsis and respiratory failure secondary to suspected pneumonia. After obtaining blood and sputum cultures, the initial empiric antibiotic coverage should be which of the following?
While visiting a neighbor, a 14-year-old girl is bitten on the left hand by the neighbor’s pet cat. The cat is an indoor pet and has had all of the required routine vaccinations. You see the girl in the office approximately 1 hour after the
injury. On the dorsum of the left hand you see two shallow puncture wounds that are not actively bleeding. She has full range of motion of her hand, normal capillary refill, and sensation. You see in the chart that the patient had a
diphtheria/tetanus (dT) booster vaccine last year. What is the most appropriate management at this time?
A 40-year-old woman presents with nausea, vomiting, and weakness. She has been amenorrheic since the birth of her last child 1 year ago and has not felt well since that time. On examination, she appears chronically ill, her thyroid is not palpable, and there is no galactorrhea. Laboratory studies on admission include:
The most appropriate next step is to start treatment with which of the following?
An 18-year-old female presents for evaluation of facial acne. On examination, she has multiple comedones, papules, and pustules on her forehead, nose, cheeks, and chin. She also has several distinct nodules, each greater than 5 mm in diameter. Which of the following is most appropriate for initial inclusion in a regimen to treat this patient’s acne?
A 34-year-old male presents with a penile lesion. Your history, physical examination, and serology confirm a diagnosis of syphilis. The patient reports that his mother told him he was “allergic” to penicillin. He does not recall any
personal history of anaphylaxis or rash to antibiotics however he has never been “sick.” How would you manage this patient?
A 55-year-old male is brought to the ED, by ambulance, because of crushing chest pain radiating to his left shoulder and arm that started 1 hour ago. He has a history of hypertension, high cholesterol, and has smoked a pack of cigarettes a day for 30 years. He has never had symptoms like this before. Which of the following would be most likely to be seen on an ECG?
A 70-year-old male is seen in the office for chest pain. He reports that he is getting substernal chest pain, without radiation, when he mows his lawn. The pain resolves with 10–15 minutes of rest. He has never had pain at rest. He has no other cardiac complaints and his review of systems is otherwise negative. He has an unremarkable medical history and takes only a baby aspirin a day. On examination, his blood pressure is 160/70, pulse 85, and respiratory rate 16. His cardiac examination is notable for a harsh, 3/6 systolic ejection murmur along the sternal border that radiates to the carotid arteries. His carotid pulsation is noted to rise slowly and is small and sustained. His lungs are clear. The remainder of his examination is normal. Subsequent workup confirms the diagnosis of critical aortic stenosis. Which of the following treatments would be most appropriate at this time?
A 19-year-old woman begins chemotherapy for an acute leukemia. Although you determine that her renal function is unimpaired prior to the initiation of treatment, you feel that she may be at high risk for development of tumor lysis syndrome given her condition’s typically good response to chemotherapy. Which of the following is typically seen as a feature of tumor lysis syndrome?
A 72-year-old man with a diagnosis of prostate cancer was recently seen in the clinic for restaging and re-evaluation. His bone scan showed development of widespread osseous metastases and his PSA was rising. He was started on leuprolide acetate, a gonadotropin releasing-hormone (GnRH) agonist. He now returns to the clinic complaining of new severe mid-thoracic back pain, which is worse with recumbency and worse with Valsalva maneuver. He also reports that he has a brief but intense electric shock sensation in his lower extremities when he bends over to tie his shoes. On physical exam, he had localized tenderness over the mid-thoracic spine, but his motor strength, sensation, and deep tendon reflexes are all intact. What is the most appropriate next step?
A 25-year-old man presents to the ER with a 3-month history of intermittent pounding headaches, sweating, and palpitations. He denies any symptoms of depression or anxiety. On examination, he is a thin gentleman, BP 240/120, heart rate 110/minute, thyroid not enlarged. There is no prior history of hypertension. The most likely diagnosis is which of the following?
A young college student is brought to your office by his fiancé for evaluation of weight loss. He tells you that, over the past few months, he seems to be unable to gain any weight despite having a ferocious appetite and that he is steadily losing weight. He has also noticed increased thirst and urination. Over the past few nights, he has awakened several times to go to the bathroom. You suspect that he may have developed diabetes. Which of the following is a diagnostic criterion for diabetes mellitus (DM)?
A 24-year-old male medical student is admitted to the hospital for the evaluation of a 3-month history of bloody stools. The patient has approximately six blood stained or blood streaked stools per day, associated with relatively little, if any, pain. He has not had any weight loss, and he has been able to attend classes without interruption. He denies any fecal incontinence. He has no prior medical history. Review of systems is remarkable only for occasional fevers and the fact that the patient quit smoking approximately 8 months ago. A colonoscopy is performed and reveals a granular, friable colonic mucosal surface with loss of normal vascular pattern from the anal verge to the hepatic flexure of the colon. Biopsies reveal prominent neutrophils in the epithelium and cryptitis with focal crypt abscesses, and no dysplasia. The patient is diagnosed with ulcerative colitis. In addition to an increased lifetime risk of colon cancer, the patient is also at increased risk for which of the following tumors?
A 28-year-old male, well known to your clinic, presents for management of swelling, pain, and tenderness that has developed in his left ankle and right
knee. It has persisted for 1 month. Your patient reports that he developed severe diarrhea after a picnic 1 month prior to the onset of his arthritis. During the interval between the diarrhea and onset of arthritis, he developed a “pink eye” that lasted for 4 days. He denies any symptoms of back pain or stiffness. You remember that he was treated with ceftriaxone and doxycycline for gonorrhea 2 years ago, which he acquired from sexual activity with multiple partners. Since that time, he has been in a monogamous relationship with his wife and has not
had any genitourinary symptoms. He promises that he has been faithful to his wife and has not engaged in unprotected sexual activity outside his marriage.
His physical examination is notable for a swollen left ankle, swollen right knee, and the absence of penile discharge or any skin lesions. The patient’s symptoms do not respond to your initial therapeutic management. You suspect that his condition is refractory to treatment. Which of the following should you consider at this time?
A 54-year-old Asian female with no significant medical history presents with frontal headache, eye pain, nausea, and vomiting. Her abdominal examination shows mild diffuse tenderness but no rebound or guarding. Her mucous membranes are dry. Her vision is blurry in both eyes, her eyes are injected but her extraocular muscles are intact. Her pupils are mid-dilated and fixed. Which of the following is appropriate initial management of this patient?
A 68-year-old White male, with a history of hypertension, an 80 pack-year history of tobacco use and emphysema, is brought into the ER because of 4 days of progressive confusion and lethargy. His wife notes that he takes amlodipine for his hypertension. He does not use over-the-counter (OTC) medications, alcohol, or drugs. Furthermore, she indicates that he has unintentionally lost approximately 30 lbs in the last 6 months. His physical examination shows that he is afebrile with a blood pressure of 142/85, heart rate of 92 (no orthostatic changes), and a room-air O2 saturation of 91%. He is 70 kg. The patient appears cachectic. He is arousable but lethargic and unable to follow any commands. His mucous membranes are moist, heart rate regular without murmurs or a S3/S4 gallop, and extremities without any edema. His pulmonary examination shows mildly diminished breath sounds in the right lower lobe with wheezing bilaterally. The patient is unable to follow commands during neurologic examination but moves all his extremities spontaneously. Laboratory results are as follows:
Which of the following is the correct statement regarding the treatment of hyponatremia?
A patient you see routinely in the clinic has elevated liver function tests. ALT is 89, AST is 75, and the total bilirubin and alkaline phosphatase are normal. The patient has no past history of hepatitis, taking medications, or excessive drinking. You order hepatitis serologies. The results are as follows: Positive: HBsAg and anti-HBc. Negative: anti-HBs, anti-HBc IgM, anti-HAV, and anti-HCV What is the most appropriate next step for this patient?
A 31-year-old female health care worker presents to your clinic after a needlestick injury from a patient who subsequently left against medical advice prior to laboratory analysis for HIV or hepatitis. You advise your colleague that:
A 29-year-old woman complains of fatigue and decreased exercise tolerance. She takes no medications and denies changes in the color of the stool. Physical examination is significant for pale skin and conjunctivae. Stool was negative for blood. Laboratory evaluation revealed Hgb of 7.8 g/dL, reticulocytopenia, microcytosis, and hypochromia. Which of the following would most likely be found on further laboratory testing?
A 32-year-old woman complains of episodic confusion in the morning for the past 6 months. During one of these episodes, she was brought to the ER and her serum glucose was found to be 40 mg/dL. She was given intravenous dextrose
and her symptoms resolved within 15 minutes. She has gained approximately 25 lbs during the past year. Which of the following would be the most appropriate next step?
An 18-year-old female presents for evaluation of facial acne. On examination, she has multiple comedones, papules, and pustules on her forehead, nose, cheeks, and chin. She also has several distinct nodules, each greater than 5 mm in diameter. After 6 months of appropriate treatment, the severity of your patient’s acne remains essentially unchanged. You rule out several potential causes for the patient’s recalcitrant acne and decide that more aggressive therapy is warranted. As you discuss this option with your patient, which of the following side effects of
the proposed treatment necessitates extensive counseling prior to initiation of therapy?
A 57-year-old male presents for annual examination. On history, you note a history of colon cancer that was successfully surgically resected 2 years ago. He had a negative postoperative colonoscopy 6 months later and another negative colonoscopy last year. Which of the following would be the most appropriate colon cancer screening regimen for this patient?
A 55-year-old male is brought to the ED, by ambulance, because of crushing chest pain radiating to his left shoulder and arm that started 1 hour ago. He has a history of hypertension, high cholesterol, and has smoked a pack of cigarettes a day for 30 years. He has never had symptoms like this before. While monitored in the ER, the patient’s rhythm suddenly converts to ventricular tachycardia, and he becomes pulseless and unresponsive. Which of the following would be the most appropriate initial management of this situation?
A 42-year-old woman with hyperlipidemia, hypertension, and hypothyroidism presents to your office for a routine follow-up visit. Her blood pressure is well controlled with hydrochlorothiazide. She has been on a stable dose of levothyroxine for 8 years and measurement of her TSH today is within normal limits. However, her LDL cholesterol level remains elevated despite taking a statin for the past 9 months and complying with lifestyle modifications. You decide that the addition of a low dose of cholestyramine would provide her with additional benefit. How would you advise the patient before beginning this therapy?
A 37-year-old White executive secretary comes to you after she found a lump in her right breast while she was showering. She describes a lesion beneath her right nipple. You question her about her personal and family history. She began
menarche at age 12, and she is still having regular menstrual periods. She has had two children; the first was born when she was 25 years old. She has no family history of breast, ovarian, or colon cancer on either her maternal or paternal side.
You perform a physical examination including a careful examination of her breasts. You note that her breasts contain many small cysts bilaterally. However, you also palpate a localized, firm, non-tender mass below the right areola. You also describe a peau d’orange appearance of the areola. What should you advise her?
A 72-year-old man with a diagnosis of prostate cancer was recently seen in the clinic for restaging and re-evaluation. His bone scan showed development of widespread osseous metastases and his PSA was rising. He was started on leuprolide acetate, a gonadotropin releasing-hormone (GnRH) agonist. He now returns to the clinic complaining of new severe mid-thoracic back pain, which is worse with recumbency and worse with Valsalva maneuver. He also reports that he has a brief but intense electric shock sensation in his lower extremities when he bends over to tie his shoes. What is the most important prognostic factor
regarding this patient’s ultimate neurological outcome?
A 40-year-old woman presents with headaches and visual disturbances for the past 6 months. She suddenly developed amenorrhea 2 years ago. One year ago, she noticed milky discharge from her left breast. The most likely diagnosis is which of the following?
A 67-year-old male with a history of type II diabetes and hypertension is hospitalized with complaints of retrosternal chest pain that radiates to the left arm and jaw. In the ED, an electrocardiogram (ECG) showed S-T segment depressions in the inferior and lateral leads. He has been given the diagnosis of acute coronary syndrome and admitted to the coronary care unit for further evaluation and treatment. Admission laboratory values reveal a total cholesterol of 270, a low-density lipoprotein (LDL) of 190, and a high-density lipoprotein (HDL) of 28. He is currently smoking a pack of cigarettes per day and lives a sedentary life. He is clearly overweight and his blood pressure, despite medication, remains elevated at 150/88. His last HgbA1C less than a month ago was 9.8%. After being discharged from the hospital, which of the following cholesterol lowering regimens should be recommended to this patient?
A 61-year-old man comes to your office for a checkup. He currently feels well and has no focal complaints. He has a past medical history significant for well-
controlled hypertension, and his gallbladder was removed 3 years ago in the setting of acute cholecystitis. He does not smoke and drinks one to two
alcoholic beverages per day. Family history is remarkable for colon cancer in his mother at age 45 and a brother at age 49. He has a sister who developed endometrial cancer at age 53. He has never undergone colon cancer screening and is interested in pursuing this. Which colorectal cancer screening test would
be best for this patient?
A 42-year-old man presents to your clinic with a 1-week history of pain and inflammation involving his right first metatarsophalangeal (MTP) joint. He
describes the pain as sudden in onset and worse at night. He denies experiencing any fever or traumatic injury to the joint and states that he has never had this type of pain before. He denies any chronic medical conditions, any prior surgery, and any current medication use. Besides an erythematous and exquisitely tender right first MTP joint, the remainder of his physical examination is unremarkable. Aspiration of the patient’s right first MTP joint space is likely to reveal which of the following?
A 64-year-old male with a history of hypertension and tobacco abuse presents for follow-up after a routine physical during which he was found to have 4–5 red blood cells (RBCs) per high-power field (HPF) on a screening urinalysis. The urinalysis was negative for leukocytes, nitrites, epithelial cells, and ketones. The patient denies any complaints and the review of systems is essentially negative. What would be your initial approach in the workup of this patient with asymptomatic microscopic hematuria?
A 53-year-old Black male, with a history of hypertension, hepatitis C, and newly diagnosed nonsmall cell lung cancer, undergoes his first round of chemotherapy, which includes cisplatin. You are called to see this patient 5 days into his hospitalization for oliguria and laboratory abnormalities. Other than the chemotherapy, he is receiving lansoprazole, acetaminophen, and an infusion of D5— 0.9% normal saline at 50 mL/h. On examination, his BP is 98/60 and heart rate is irregular, between 40 and 50 bpm. His physical examination shows a middle-aged male in no acute distress. His cardiac examination is unremarkable, his lungs show bibasilar crackles, and the abdominal examination is positive for a palpable spleen tip without any hepatomegaly or abdominal tenderness. He has
trace bilateral ankle edema. His distal pulses are irregular. The neurologic examination was unremarkable. His laboratory (serum sample) results are as follows:
Which electrolyte/acid-base abnormality is most likely responsible for the finding